Study IDs Immune Abnormality Possibly Causing Long COVID

Article Type
Changed
Fri, 02/23/2024 - 11:39

Swiss scientists have identified immune system abnormalities in patients with long COVID that might open the door to new diagnostic tests and treatments.

The researchers found that a group of proteins in the blood that are part of the body’s immune response called the “complement system” are not working properly in patients with long COVID.

Blood samples turned up important differences between those who recovered from COVID and those who did not. These differences might be used as biomarkers to diagnose long COVID and might even point the way to new treatments for the condition, the researchers said.

By testing for 6500 blood proteins in about 300 patients, the Swiss researchers found that dysfunctional complement system proteins could possibly explain fatigue and “smoldering inflammation,” said Onur Boyman, MD, a professor of immunology from University Hospital Zurich in Zurich, Switzerland.

Long COVID has been linked to hundreds of symptoms including brain fog, chronic fatigue, pain, and digestive issues. Various factors drive the condition and likely work with one another other, said David Putrino, PhD, from the Icahn School of Medicine at Mount Sinai in New York City. The Swiss study is useful because “we’re trying to best understand how we can explain all of this far-reaching pathobiology,” he said.
 

Testing Across Continents

Dr. Boyman’s team collected blood samples from people with COVID in Europe and New York and tracked them. They compared those who developed long COVID with those who did not. One protein that was most unique to patients with long COVID is a blood complement that activates the immune system, Dr. Boyman said. But in people with long COVID, the immune response stays activated after the virus is gone. He described the response as “smoldering inflammation” in multiple organs, including the lungs and the gastrointestinal system.

The complement system also plays a role in clearing the body of dead cells. If the cells “lie around too much,” they can trigger an immune response, he said.

That may explain exercise intolerance in people with long COVID, Dr. Boyman said. Some people with long COVID have inflammation in the epithelium — the inner layer of their blood vessels. This would make it harder for the circulatory systems to recover from exercise, Dr. Boyman said.

“We think this regulated complement system is actually quite a central piece of the puzzle,” he said.
 

The Microclot Connection

The findings also support past research linking blood clots to long COVID. He suggested that clinicians and researchers consider testing drugs that regulate or inhibit the complementary system as a treatment of long COVID. Dr. Boyman said they are currently used for rare immune diseases.

Resia Pretorius, PhD, a professor of physiological sciences at Stellenbosch University in Stellenbosch, South Africa, said scientists studying the role of microclots in patients with long COVID often see complementary proteins inside the clots, so it has already been associated with long COVID. But she likened this clotting process to a garbage can that “just rolls along and collects everything that gets in its way. I think they are actively driving inflammation and disease.”

One factor complicating long COVID diagnosis and treatment is that it is a complex condition that involves multiple organ systems. That’s why the latest research suggests an underlying driver for the multiple symptoms of long COVID, Dr. Putrino said.

“Not every person has every symptom; not every person has every organ system affected,” Dr. Putrino said. “Whatever is happening is decided across the whole body.”
 

 

 

Research Offers New Direction

The Swiss paper contributes to the effort to identify systemic issues contributing to long COVID. It gives researchers one more thing to test for and link to specific, long COVID symptoms, opening the door to new treatments, Dr. Putrino said.

He doesn’t think the study supports treating the complement dysfunction if researchers don’t know what’s driving it. It may be complicated by the body’s failure to clear the virus completely, he said.

Dr. Pretorius recommended doctors test patients with long COVID for specific symptoms that may be treated using existing therapies. “If you think your patient had vascular pathology, you can test for it,” she said.

Some patients have found certain supplements and over-the-counter products helpful, she said. Among them: Coenzyme Q 10 and clot-busters such as streptokinase and Nattokinase (though she noted some doctors may not be comfortable with supplements).

“It’s the only thing we have until we’ve got trials,” she said.

Dr. Putrino said more research is needed to identify potential root causes and symptoms. A common refrain, but the only thing that will lead to specific treatments.

A version of this article appeared on Medscape.com.

Publications
Topics
Sections

Swiss scientists have identified immune system abnormalities in patients with long COVID that might open the door to new diagnostic tests and treatments.

The researchers found that a group of proteins in the blood that are part of the body’s immune response called the “complement system” are not working properly in patients with long COVID.

Blood samples turned up important differences between those who recovered from COVID and those who did not. These differences might be used as biomarkers to diagnose long COVID and might even point the way to new treatments for the condition, the researchers said.

By testing for 6500 blood proteins in about 300 patients, the Swiss researchers found that dysfunctional complement system proteins could possibly explain fatigue and “smoldering inflammation,” said Onur Boyman, MD, a professor of immunology from University Hospital Zurich in Zurich, Switzerland.

Long COVID has been linked to hundreds of symptoms including brain fog, chronic fatigue, pain, and digestive issues. Various factors drive the condition and likely work with one another other, said David Putrino, PhD, from the Icahn School of Medicine at Mount Sinai in New York City. The Swiss study is useful because “we’re trying to best understand how we can explain all of this far-reaching pathobiology,” he said.
 

Testing Across Continents

Dr. Boyman’s team collected blood samples from people with COVID in Europe and New York and tracked them. They compared those who developed long COVID with those who did not. One protein that was most unique to patients with long COVID is a blood complement that activates the immune system, Dr. Boyman said. But in people with long COVID, the immune response stays activated after the virus is gone. He described the response as “smoldering inflammation” in multiple organs, including the lungs and the gastrointestinal system.

The complement system also plays a role in clearing the body of dead cells. If the cells “lie around too much,” they can trigger an immune response, he said.

That may explain exercise intolerance in people with long COVID, Dr. Boyman said. Some people with long COVID have inflammation in the epithelium — the inner layer of their blood vessels. This would make it harder for the circulatory systems to recover from exercise, Dr. Boyman said.

“We think this regulated complement system is actually quite a central piece of the puzzle,” he said.
 

The Microclot Connection

The findings also support past research linking blood clots to long COVID. He suggested that clinicians and researchers consider testing drugs that regulate or inhibit the complementary system as a treatment of long COVID. Dr. Boyman said they are currently used for rare immune diseases.

Resia Pretorius, PhD, a professor of physiological sciences at Stellenbosch University in Stellenbosch, South Africa, said scientists studying the role of microclots in patients with long COVID often see complementary proteins inside the clots, so it has already been associated with long COVID. But she likened this clotting process to a garbage can that “just rolls along and collects everything that gets in its way. I think they are actively driving inflammation and disease.”

One factor complicating long COVID diagnosis and treatment is that it is a complex condition that involves multiple organ systems. That’s why the latest research suggests an underlying driver for the multiple symptoms of long COVID, Dr. Putrino said.

“Not every person has every symptom; not every person has every organ system affected,” Dr. Putrino said. “Whatever is happening is decided across the whole body.”
 

 

 

Research Offers New Direction

The Swiss paper contributes to the effort to identify systemic issues contributing to long COVID. It gives researchers one more thing to test for and link to specific, long COVID symptoms, opening the door to new treatments, Dr. Putrino said.

He doesn’t think the study supports treating the complement dysfunction if researchers don’t know what’s driving it. It may be complicated by the body’s failure to clear the virus completely, he said.

Dr. Pretorius recommended doctors test patients with long COVID for specific symptoms that may be treated using existing therapies. “If you think your patient had vascular pathology, you can test for it,” she said.

Some patients have found certain supplements and over-the-counter products helpful, she said. Among them: Coenzyme Q 10 and clot-busters such as streptokinase and Nattokinase (though she noted some doctors may not be comfortable with supplements).

“It’s the only thing we have until we’ve got trials,” she said.

Dr. Putrino said more research is needed to identify potential root causes and symptoms. A common refrain, but the only thing that will lead to specific treatments.

A version of this article appeared on Medscape.com.

Swiss scientists have identified immune system abnormalities in patients with long COVID that might open the door to new diagnostic tests and treatments.

The researchers found that a group of proteins in the blood that are part of the body’s immune response called the “complement system” are not working properly in patients with long COVID.

Blood samples turned up important differences between those who recovered from COVID and those who did not. These differences might be used as biomarkers to diagnose long COVID and might even point the way to new treatments for the condition, the researchers said.

By testing for 6500 blood proteins in about 300 patients, the Swiss researchers found that dysfunctional complement system proteins could possibly explain fatigue and “smoldering inflammation,” said Onur Boyman, MD, a professor of immunology from University Hospital Zurich in Zurich, Switzerland.

Long COVID has been linked to hundreds of symptoms including brain fog, chronic fatigue, pain, and digestive issues. Various factors drive the condition and likely work with one another other, said David Putrino, PhD, from the Icahn School of Medicine at Mount Sinai in New York City. The Swiss study is useful because “we’re trying to best understand how we can explain all of this far-reaching pathobiology,” he said.
 

Testing Across Continents

Dr. Boyman’s team collected blood samples from people with COVID in Europe and New York and tracked them. They compared those who developed long COVID with those who did not. One protein that was most unique to patients with long COVID is a blood complement that activates the immune system, Dr. Boyman said. But in people with long COVID, the immune response stays activated after the virus is gone. He described the response as “smoldering inflammation” in multiple organs, including the lungs and the gastrointestinal system.

The complement system also plays a role in clearing the body of dead cells. If the cells “lie around too much,” they can trigger an immune response, he said.

That may explain exercise intolerance in people with long COVID, Dr. Boyman said. Some people with long COVID have inflammation in the epithelium — the inner layer of their blood vessels. This would make it harder for the circulatory systems to recover from exercise, Dr. Boyman said.

“We think this regulated complement system is actually quite a central piece of the puzzle,” he said.
 

The Microclot Connection

The findings also support past research linking blood clots to long COVID. He suggested that clinicians and researchers consider testing drugs that regulate or inhibit the complementary system as a treatment of long COVID. Dr. Boyman said they are currently used for rare immune diseases.

Resia Pretorius, PhD, a professor of physiological sciences at Stellenbosch University in Stellenbosch, South Africa, said scientists studying the role of microclots in patients with long COVID often see complementary proteins inside the clots, so it has already been associated with long COVID. But she likened this clotting process to a garbage can that “just rolls along and collects everything that gets in its way. I think they are actively driving inflammation and disease.”

One factor complicating long COVID diagnosis and treatment is that it is a complex condition that involves multiple organ systems. That’s why the latest research suggests an underlying driver for the multiple symptoms of long COVID, Dr. Putrino said.

“Not every person has every symptom; not every person has every organ system affected,” Dr. Putrino said. “Whatever is happening is decided across the whole body.”
 

 

 

Research Offers New Direction

The Swiss paper contributes to the effort to identify systemic issues contributing to long COVID. It gives researchers one more thing to test for and link to specific, long COVID symptoms, opening the door to new treatments, Dr. Putrino said.

He doesn’t think the study supports treating the complement dysfunction if researchers don’t know what’s driving it. It may be complicated by the body’s failure to clear the virus completely, he said.

Dr. Pretorius recommended doctors test patients with long COVID for specific symptoms that may be treated using existing therapies. “If you think your patient had vascular pathology, you can test for it,” she said.

Some patients have found certain supplements and over-the-counter products helpful, she said. Among them: Coenzyme Q 10 and clot-busters such as streptokinase and Nattokinase (though she noted some doctors may not be comfortable with supplements).

“It’s the only thing we have until we’ve got trials,” she said.

Dr. Putrino said more research is needed to identify potential root causes and symptoms. A common refrain, but the only thing that will lead to specific treatments.

A version of this article appeared on Medscape.com.

Publications
Publications
Topics
Article Type
Sections
Disallow All Ads
Content Gating
No Gating (article Unlocked/Free)
Alternative CME
Disqus Comments
Default
Use ProPublica
Hide sidebar & use full width
render the right sidebar.
Conference Recap Checkbox
Not Conference Recap
Clinical Edge
Display the Slideshow in this Article
Medscape Article
Display survey writer
Reuters content
Disable Inline Native ads
WebMD Article

Can Gargling With Mouthwash Help Manage Type 2 Diabetes?

Article Type
Changed
Wed, 02/21/2024 - 22:12

 

TOPLINE:

Gargling with mouthwash two to three times a day can reduce periodontopathic bacteria and possibly improve glycemic control in people with type 2 diabetes (T2D), especially younger adults.

METHODOLOGY:

  • A total of 173 patients with T2D who had at least six total periodontopathic bacteria in their mouths and  ≥ 6.5% were instructed to gargle with water three times a day for 6 months, followed by gargling with chlorhexidine gluconate mouthwash three times a day for the next 6 months.
  • Saliva specimens were collected every 1-2 months at clinic visits totaling 6-12 samples per study period and bacterial DNA examined for three red complex species, namely, Porphyromonas gingivalis, Treponema denticola, and Tannerella forsythia.

TAKEAWAY:

  • Twelve individuals who gargled once a day or less showed no significant reductions in red complex species after mouthwash or water gargling.
  • By contrast, significant decreases in red complex bacteria were seen after 6 months of mouthwash gargling (P < .001) in the 80 who gargled twice a day and the 81 who did so three times a day compared with no changes after water gargling.
  • Among the 161 individuals who gargled at least twice a day, the decrease in red species with mouthwash vs water gargling was highly significant (P < .0001).
  • After adjustment for A1c seasonal variation, neither water gargling nor mouthwash gargling led to significant overall reduction in A1c levels.
  • However, A1c levels were significantly lower in the 83 individuals aged ≤ 68 years than among the 78 aged ≥ 69 years after gargling with mouthwash (P < .05), with no change in either group after water gargling.
  • Similarly, A1c levels were significantly reduced (P < .05) after mouthwash in the 69 with baseline A1c ≥ 7.5% compared with the 92 whose baseline A1c levels were ≤ 7.4%, with no changes in either after water.

IN PRACTICE:

“A bidirectional relationship between periodontitis and T2D has been reported. Patients with T2D are more susceptible to severe periodontitis than subjects without diabetes, and inflammatory periodontitis aggravates hyperglycemia, leading to inadequate glycemic control.” “Recently, it has been reported that patients with T2D treated for periodontitis have reduced periodontopathic bacteria and improved glycemic control. Patients with T2D complicated by periodontitis have more red complex species, and poor glycemic control is thought to be associated with increased levels of red complex species in the oral cavity.” “Further studies should be planned, taking into account various patient factors to determine the effect of mouthwash gargling on the amount of red complex species and A1c levels in patients with T2D.”

SOURCE:

This study was conducted by Saaya Matayoshi, of the Joint Research Laboratory of Science for Oral and Systemic Connection, Osaka University Graduate School of Dentistry, Osaka, Japan, and colleagues and published in Scientific Reports.

LIMITATIONS:

Only polymerase chain reaction used to detect periodontopathic bacteria so not quantified. No assessment of periodontal pocket depth. Saliva sampling conditions not standardized. Study conducted during COVID-19 pandemic; all patients wore masks. Heterogeneity in patient responses to the mouthwash.

DISCLOSURES:

This work was supported by the Fund for Scientific Promotion of Weltec Corp, Osaka, Japan. The authors declared no competing interests.

A version of this article appeared on Medscape.com.

Publications
Topics
Sections

 

TOPLINE:

Gargling with mouthwash two to three times a day can reduce periodontopathic bacteria and possibly improve glycemic control in people with type 2 diabetes (T2D), especially younger adults.

METHODOLOGY:

  • A total of 173 patients with T2D who had at least six total periodontopathic bacteria in their mouths and  ≥ 6.5% were instructed to gargle with water three times a day for 6 months, followed by gargling with chlorhexidine gluconate mouthwash three times a day for the next 6 months.
  • Saliva specimens were collected every 1-2 months at clinic visits totaling 6-12 samples per study period and bacterial DNA examined for three red complex species, namely, Porphyromonas gingivalis, Treponema denticola, and Tannerella forsythia.

TAKEAWAY:

  • Twelve individuals who gargled once a day or less showed no significant reductions in red complex species after mouthwash or water gargling.
  • By contrast, significant decreases in red complex bacteria were seen after 6 months of mouthwash gargling (P < .001) in the 80 who gargled twice a day and the 81 who did so three times a day compared with no changes after water gargling.
  • Among the 161 individuals who gargled at least twice a day, the decrease in red species with mouthwash vs water gargling was highly significant (P < .0001).
  • After adjustment for A1c seasonal variation, neither water gargling nor mouthwash gargling led to significant overall reduction in A1c levels.
  • However, A1c levels were significantly lower in the 83 individuals aged ≤ 68 years than among the 78 aged ≥ 69 years after gargling with mouthwash (P < .05), with no change in either group after water gargling.
  • Similarly, A1c levels were significantly reduced (P < .05) after mouthwash in the 69 with baseline A1c ≥ 7.5% compared with the 92 whose baseline A1c levels were ≤ 7.4%, with no changes in either after water.

IN PRACTICE:

“A bidirectional relationship between periodontitis and T2D has been reported. Patients with T2D are more susceptible to severe periodontitis than subjects without diabetes, and inflammatory periodontitis aggravates hyperglycemia, leading to inadequate glycemic control.” “Recently, it has been reported that patients with T2D treated for periodontitis have reduced periodontopathic bacteria and improved glycemic control. Patients with T2D complicated by periodontitis have more red complex species, and poor glycemic control is thought to be associated with increased levels of red complex species in the oral cavity.” “Further studies should be planned, taking into account various patient factors to determine the effect of mouthwash gargling on the amount of red complex species and A1c levels in patients with T2D.”

SOURCE:

This study was conducted by Saaya Matayoshi, of the Joint Research Laboratory of Science for Oral and Systemic Connection, Osaka University Graduate School of Dentistry, Osaka, Japan, and colleagues and published in Scientific Reports.

LIMITATIONS:

Only polymerase chain reaction used to detect periodontopathic bacteria so not quantified. No assessment of periodontal pocket depth. Saliva sampling conditions not standardized. Study conducted during COVID-19 pandemic; all patients wore masks. Heterogeneity in patient responses to the mouthwash.

DISCLOSURES:

This work was supported by the Fund for Scientific Promotion of Weltec Corp, Osaka, Japan. The authors declared no competing interests.

A version of this article appeared on Medscape.com.

 

TOPLINE:

Gargling with mouthwash two to three times a day can reduce periodontopathic bacteria and possibly improve glycemic control in people with type 2 diabetes (T2D), especially younger adults.

METHODOLOGY:

  • A total of 173 patients with T2D who had at least six total periodontopathic bacteria in their mouths and  ≥ 6.5% were instructed to gargle with water three times a day for 6 months, followed by gargling with chlorhexidine gluconate mouthwash three times a day for the next 6 months.
  • Saliva specimens were collected every 1-2 months at clinic visits totaling 6-12 samples per study period and bacterial DNA examined for three red complex species, namely, Porphyromonas gingivalis, Treponema denticola, and Tannerella forsythia.

TAKEAWAY:

  • Twelve individuals who gargled once a day or less showed no significant reductions in red complex species after mouthwash or water gargling.
  • By contrast, significant decreases in red complex bacteria were seen after 6 months of mouthwash gargling (P < .001) in the 80 who gargled twice a day and the 81 who did so three times a day compared with no changes after water gargling.
  • Among the 161 individuals who gargled at least twice a day, the decrease in red species with mouthwash vs water gargling was highly significant (P < .0001).
  • After adjustment for A1c seasonal variation, neither water gargling nor mouthwash gargling led to significant overall reduction in A1c levels.
  • However, A1c levels were significantly lower in the 83 individuals aged ≤ 68 years than among the 78 aged ≥ 69 years after gargling with mouthwash (P < .05), with no change in either group after water gargling.
  • Similarly, A1c levels were significantly reduced (P < .05) after mouthwash in the 69 with baseline A1c ≥ 7.5% compared with the 92 whose baseline A1c levels were ≤ 7.4%, with no changes in either after water.

IN PRACTICE:

“A bidirectional relationship between periodontitis and T2D has been reported. Patients with T2D are more susceptible to severe periodontitis than subjects without diabetes, and inflammatory periodontitis aggravates hyperglycemia, leading to inadequate glycemic control.” “Recently, it has been reported that patients with T2D treated for periodontitis have reduced periodontopathic bacteria and improved glycemic control. Patients with T2D complicated by periodontitis have more red complex species, and poor glycemic control is thought to be associated with increased levels of red complex species in the oral cavity.” “Further studies should be planned, taking into account various patient factors to determine the effect of mouthwash gargling on the amount of red complex species and A1c levels in patients with T2D.”

SOURCE:

This study was conducted by Saaya Matayoshi, of the Joint Research Laboratory of Science for Oral and Systemic Connection, Osaka University Graduate School of Dentistry, Osaka, Japan, and colleagues and published in Scientific Reports.

LIMITATIONS:

Only polymerase chain reaction used to detect periodontopathic bacteria so not quantified. No assessment of periodontal pocket depth. Saliva sampling conditions not standardized. Study conducted during COVID-19 pandemic; all patients wore masks. Heterogeneity in patient responses to the mouthwash.

DISCLOSURES:

This work was supported by the Fund for Scientific Promotion of Weltec Corp, Osaka, Japan. The authors declared no competing interests.

A version of this article appeared on Medscape.com.

Publications
Publications
Topics
Article Type
Sections
Disallow All Ads
Content Gating
No Gating (article Unlocked/Free)
Alternative CME
Disqus Comments
Default
Use ProPublica
Hide sidebar & use full width
render the right sidebar.
Conference Recap Checkbox
Not Conference Recap
Clinical Edge
Display the Slideshow in this Article
Medscape Article
Display survey writer
Reuters content
Disable Inline Native ads
WebMD Article

New Marker of Cardiovascular Risk Discovered in T2D

Article Type
Changed
Tue, 02/20/2024 - 13:10

A significant quantity of dysfunctional monocytes appears to indicate poor cardiovascular prognosis in patients with type 2 diabetes, according to a new publication. Nicolas Venteclef, PhD, director of an Inserm institute for diabetes research at Necker Enfants Malades Hospital in Paris, France, led the research.

Quantifying Inflammation

Patients with type 2 diabetes have about twice the risk for a cardiovascular event associated with atherosclerosis, such as a heart attack or stroke, during their lifetimes. “Predicting these complications in diabetic patients is usually very difficult,” Dr. Venteclef told this news organization.

“They are strongly associated with inflammation in these patients. Therefore, we sought to quantify this inflammation in the blood.” To do this, his team focused on monocytes, a category of white blood cells circulating in the blood. They measured the blood concentration of monocytes and the subtypes present in patients with type 2 diabetes.

The results were published in Circulation Research.
 

Dysfunctional Monocytes

The team worked with three cohorts of patients. The first, named AngioSafe-2, consisting of 672 patients with type 2 diabetes, was recruited from the diabetology departments of Lariboisière and Bichat Claude Bernard hospitals in France. This cohort allowed researchers to demonstrate that the higher the number of circulating monocytes, the greater the risk for cardiovascular events, independent of age and duration of diabetes. This observation was confirmed through a second cohort, GLUTADIAB, that comprised 279 patients with type 2 diabetes. Scientists complemented their work with molecular analysis of circulating monocytes in these two cohorts, which revealed certain predominant monocyte subtypes in patients with type 2 diabetes at high cardiovascular risk. “These monocytes are dysfunctional because they have a mitochondrial problem,” Dr. Venteclef explained.

To better understand how these results could be used to predict cardiovascular risk, the team collaborated with colleagues from the University Hospital of Nantes on a cohort called SURDIAGENE, which included 757 patients with type 2 diabetes. “We conducted a longitudinal study by following these patients for 10 years and quantifying cardiovascular events and deaths,” said Dr. Venteclef. Circulating monocyte levels were correlated with the occurrence of heart attacks or strokes. The researchers observed that patients with type 2 diabetes with a monocyte count above a certain threshold (0.5 × 109/L) had a five- to seven-times higher risk for cardiovascular events over 10 years than those with a monocyte count below this threshold.

A patent was filed at the end of 2023 to protect this discovery. “Our next step is to develop a sensor to quantify monocytes more easily and avoid blood draws,” said Dr. Venteclef. “As part of a European project, we will also launch a trial with an anti-inflammatory drug in diabetics, with the hope of interrupting the inflammatory trajectory and preventing complications.”
 

This story was translated from the Medscape French edition using several editorial tools, including AI, as part of the process. Human editors reviewed this content before publication. A version of this article appeared on Medscape.com.

Publications
Topics
Sections

A significant quantity of dysfunctional monocytes appears to indicate poor cardiovascular prognosis in patients with type 2 diabetes, according to a new publication. Nicolas Venteclef, PhD, director of an Inserm institute for diabetes research at Necker Enfants Malades Hospital in Paris, France, led the research.

Quantifying Inflammation

Patients with type 2 diabetes have about twice the risk for a cardiovascular event associated with atherosclerosis, such as a heart attack or stroke, during their lifetimes. “Predicting these complications in diabetic patients is usually very difficult,” Dr. Venteclef told this news organization.

“They are strongly associated with inflammation in these patients. Therefore, we sought to quantify this inflammation in the blood.” To do this, his team focused on monocytes, a category of white blood cells circulating in the blood. They measured the blood concentration of monocytes and the subtypes present in patients with type 2 diabetes.

The results were published in Circulation Research.
 

Dysfunctional Monocytes

The team worked with three cohorts of patients. The first, named AngioSafe-2, consisting of 672 patients with type 2 diabetes, was recruited from the diabetology departments of Lariboisière and Bichat Claude Bernard hospitals in France. This cohort allowed researchers to demonstrate that the higher the number of circulating monocytes, the greater the risk for cardiovascular events, independent of age and duration of diabetes. This observation was confirmed through a second cohort, GLUTADIAB, that comprised 279 patients with type 2 diabetes. Scientists complemented their work with molecular analysis of circulating monocytes in these two cohorts, which revealed certain predominant monocyte subtypes in patients with type 2 diabetes at high cardiovascular risk. “These monocytes are dysfunctional because they have a mitochondrial problem,” Dr. Venteclef explained.

To better understand how these results could be used to predict cardiovascular risk, the team collaborated with colleagues from the University Hospital of Nantes on a cohort called SURDIAGENE, which included 757 patients with type 2 diabetes. “We conducted a longitudinal study by following these patients for 10 years and quantifying cardiovascular events and deaths,” said Dr. Venteclef. Circulating monocyte levels were correlated with the occurrence of heart attacks or strokes. The researchers observed that patients with type 2 diabetes with a monocyte count above a certain threshold (0.5 × 109/L) had a five- to seven-times higher risk for cardiovascular events over 10 years than those with a monocyte count below this threshold.

A patent was filed at the end of 2023 to protect this discovery. “Our next step is to develop a sensor to quantify monocytes more easily and avoid blood draws,” said Dr. Venteclef. “As part of a European project, we will also launch a trial with an anti-inflammatory drug in diabetics, with the hope of interrupting the inflammatory trajectory and preventing complications.”
 

This story was translated from the Medscape French edition using several editorial tools, including AI, as part of the process. Human editors reviewed this content before publication. A version of this article appeared on Medscape.com.

A significant quantity of dysfunctional monocytes appears to indicate poor cardiovascular prognosis in patients with type 2 diabetes, according to a new publication. Nicolas Venteclef, PhD, director of an Inserm institute for diabetes research at Necker Enfants Malades Hospital in Paris, France, led the research.

Quantifying Inflammation

Patients with type 2 diabetes have about twice the risk for a cardiovascular event associated with atherosclerosis, such as a heart attack or stroke, during their lifetimes. “Predicting these complications in diabetic patients is usually very difficult,” Dr. Venteclef told this news organization.

“They are strongly associated with inflammation in these patients. Therefore, we sought to quantify this inflammation in the blood.” To do this, his team focused on monocytes, a category of white blood cells circulating in the blood. They measured the blood concentration of monocytes and the subtypes present in patients with type 2 diabetes.

The results were published in Circulation Research.
 

Dysfunctional Monocytes

The team worked with three cohorts of patients. The first, named AngioSafe-2, consisting of 672 patients with type 2 diabetes, was recruited from the diabetology departments of Lariboisière and Bichat Claude Bernard hospitals in France. This cohort allowed researchers to demonstrate that the higher the number of circulating monocytes, the greater the risk for cardiovascular events, independent of age and duration of diabetes. This observation was confirmed through a second cohort, GLUTADIAB, that comprised 279 patients with type 2 diabetes. Scientists complemented their work with molecular analysis of circulating monocytes in these two cohorts, which revealed certain predominant monocyte subtypes in patients with type 2 diabetes at high cardiovascular risk. “These monocytes are dysfunctional because they have a mitochondrial problem,” Dr. Venteclef explained.

To better understand how these results could be used to predict cardiovascular risk, the team collaborated with colleagues from the University Hospital of Nantes on a cohort called SURDIAGENE, which included 757 patients with type 2 diabetes. “We conducted a longitudinal study by following these patients for 10 years and quantifying cardiovascular events and deaths,” said Dr. Venteclef. Circulating monocyte levels were correlated with the occurrence of heart attacks or strokes. The researchers observed that patients with type 2 diabetes with a monocyte count above a certain threshold (0.5 × 109/L) had a five- to seven-times higher risk for cardiovascular events over 10 years than those with a monocyte count below this threshold.

A patent was filed at the end of 2023 to protect this discovery. “Our next step is to develop a sensor to quantify monocytes more easily and avoid blood draws,” said Dr. Venteclef. “As part of a European project, we will also launch a trial with an anti-inflammatory drug in diabetics, with the hope of interrupting the inflammatory trajectory and preventing complications.”
 

This story was translated from the Medscape French edition using several editorial tools, including AI, as part of the process. Human editors reviewed this content before publication. A version of this article appeared on Medscape.com.

Publications
Publications
Topics
Article Type
Sections
Disallow All Ads
Content Gating
No Gating (article Unlocked/Free)
Alternative CME
Disqus Comments
Default
Use ProPublica
Hide sidebar & use full width
render the right sidebar.
Conference Recap Checkbox
Not Conference Recap
Clinical Edge
Display the Slideshow in this Article
Medscape Article
Display survey writer
Reuters content
Disable Inline Native ads
WebMD Article

Can a Plant-Based Diet Lower Type 2 Diabetes Risk?

Article Type
Changed
Tue, 02/20/2024 - 13:36

 

TOPLINE:

Greater adherence to a plant-based dietary pattern was associated with a lower risk of developing type 2 diabetes (T2D) among middle-aged US adults. Greater intake of healthful plant foods, rather than lower intake of non-red meat animal foods, was the main factor underlying the inverse associations.

METHODOLOGY:

  • The study population was 11,965 adults aged 45-64 years from the Atherosclerosis Risk in Communities (ARIC) study who didn›t have diabetes at baseline and who completed food-frequency questionnaires.
  • Plant-based diet adherence was classified overall with the plant-based diet index (PDI) and also with higher healthful PDI (hPDI) and higher unhealthful PDI (uPDI) indexes.

TAKEAWAY:

  • Mean daily total plant and animal food intakes for the highest quintile (5) were 15.1 and 3.4 servings per day, respectively, whereas average consumption for the lowest quintile (1) was 9.9 and 5.8 servings per day, respectively.
  • During a median 22 years’ follow-up, 35% (n = 4208) of the participants developed T2D.
  • After controlling for age, sex, race center, energy intake, education, income, smoking, alcohol intake, physical activity, and margarine intake, those in PDI quintile 5 had a significantly lower risk of developing T2D than in quintile 1 (hazard ratio, 0.89; P = .01).
  • As a continuous score, each 10-point higher PDI score was associated with a significant 6% lower risk for T2D (P = .01).
  • Higher hPDI scores were also inversely associated with T2D risk (hazard ratio, 0.85 for quintiles 5 vs 1; P < .001), and (0.90 per each 10 units higher; P < .001).
  • Higher uPDI scores were not significantly associated with diabetes risk, regardless of adjustments (P > .05).
  • Associations between plant-based diet scores and diabetes did not differ by sex, age, race, or body mass index (BMI) after accounting for multiple comparisons (all P interaction > .05).
  • Further adjustment for BMI attenuated the associations between overall and healthy plant-based diets and diabetes risk, suggesting that lower adiposity may partly explain the favorable association.

IN PRACTICE:

“Emphasizing plant foods may be an effective dietary strategy to delay or prevent the onset of diabetes.”

SOURCE:

The study conducted by Valerie K. Sullivan, PhD, RD, of the Welch Center for Prevention, Epidemiology, and Clinical Research, Johns Hopkins University, Baltimore, Maryland, and colleagues was published online in Diabetes Care.

LIMITATIONS:

The limitations were self-reported dietary intake, diets assessed decades ago, possible food misclassification, possible selection bias, and residual confounding.

DISCLOSURES:

The ARIC study was funded by the US National Institutes of Health. The authors had no further disclosures.

A version of this article appeared on Medscape.com.

Publications
Topics
Sections

 

TOPLINE:

Greater adherence to a plant-based dietary pattern was associated with a lower risk of developing type 2 diabetes (T2D) among middle-aged US adults. Greater intake of healthful plant foods, rather than lower intake of non-red meat animal foods, was the main factor underlying the inverse associations.

METHODOLOGY:

  • The study population was 11,965 adults aged 45-64 years from the Atherosclerosis Risk in Communities (ARIC) study who didn›t have diabetes at baseline and who completed food-frequency questionnaires.
  • Plant-based diet adherence was classified overall with the plant-based diet index (PDI) and also with higher healthful PDI (hPDI) and higher unhealthful PDI (uPDI) indexes.

TAKEAWAY:

  • Mean daily total plant and animal food intakes for the highest quintile (5) were 15.1 and 3.4 servings per day, respectively, whereas average consumption for the lowest quintile (1) was 9.9 and 5.8 servings per day, respectively.
  • During a median 22 years’ follow-up, 35% (n = 4208) of the participants developed T2D.
  • After controlling for age, sex, race center, energy intake, education, income, smoking, alcohol intake, physical activity, and margarine intake, those in PDI quintile 5 had a significantly lower risk of developing T2D than in quintile 1 (hazard ratio, 0.89; P = .01).
  • As a continuous score, each 10-point higher PDI score was associated with a significant 6% lower risk for T2D (P = .01).
  • Higher hPDI scores were also inversely associated with T2D risk (hazard ratio, 0.85 for quintiles 5 vs 1; P < .001), and (0.90 per each 10 units higher; P < .001).
  • Higher uPDI scores were not significantly associated with diabetes risk, regardless of adjustments (P > .05).
  • Associations between plant-based diet scores and diabetes did not differ by sex, age, race, or body mass index (BMI) after accounting for multiple comparisons (all P interaction > .05).
  • Further adjustment for BMI attenuated the associations between overall and healthy plant-based diets and diabetes risk, suggesting that lower adiposity may partly explain the favorable association.

IN PRACTICE:

“Emphasizing plant foods may be an effective dietary strategy to delay or prevent the onset of diabetes.”

SOURCE:

The study conducted by Valerie K. Sullivan, PhD, RD, of the Welch Center for Prevention, Epidemiology, and Clinical Research, Johns Hopkins University, Baltimore, Maryland, and colleagues was published online in Diabetes Care.

LIMITATIONS:

The limitations were self-reported dietary intake, diets assessed decades ago, possible food misclassification, possible selection bias, and residual confounding.

DISCLOSURES:

The ARIC study was funded by the US National Institutes of Health. The authors had no further disclosures.

A version of this article appeared on Medscape.com.

 

TOPLINE:

Greater adherence to a plant-based dietary pattern was associated with a lower risk of developing type 2 diabetes (T2D) among middle-aged US adults. Greater intake of healthful plant foods, rather than lower intake of non-red meat animal foods, was the main factor underlying the inverse associations.

METHODOLOGY:

  • The study population was 11,965 adults aged 45-64 years from the Atherosclerosis Risk in Communities (ARIC) study who didn›t have diabetes at baseline and who completed food-frequency questionnaires.
  • Plant-based diet adherence was classified overall with the plant-based diet index (PDI) and also with higher healthful PDI (hPDI) and higher unhealthful PDI (uPDI) indexes.

TAKEAWAY:

  • Mean daily total plant and animal food intakes for the highest quintile (5) were 15.1 and 3.4 servings per day, respectively, whereas average consumption for the lowest quintile (1) was 9.9 and 5.8 servings per day, respectively.
  • During a median 22 years’ follow-up, 35% (n = 4208) of the participants developed T2D.
  • After controlling for age, sex, race center, energy intake, education, income, smoking, alcohol intake, physical activity, and margarine intake, those in PDI quintile 5 had a significantly lower risk of developing T2D than in quintile 1 (hazard ratio, 0.89; P = .01).
  • As a continuous score, each 10-point higher PDI score was associated with a significant 6% lower risk for T2D (P = .01).
  • Higher hPDI scores were also inversely associated with T2D risk (hazard ratio, 0.85 for quintiles 5 vs 1; P < .001), and (0.90 per each 10 units higher; P < .001).
  • Higher uPDI scores were not significantly associated with diabetes risk, regardless of adjustments (P > .05).
  • Associations between plant-based diet scores and diabetes did not differ by sex, age, race, or body mass index (BMI) after accounting for multiple comparisons (all P interaction > .05).
  • Further adjustment for BMI attenuated the associations between overall and healthy plant-based diets and diabetes risk, suggesting that lower adiposity may partly explain the favorable association.

IN PRACTICE:

“Emphasizing plant foods may be an effective dietary strategy to delay or prevent the onset of diabetes.”

SOURCE:

The study conducted by Valerie K. Sullivan, PhD, RD, of the Welch Center for Prevention, Epidemiology, and Clinical Research, Johns Hopkins University, Baltimore, Maryland, and colleagues was published online in Diabetes Care.

LIMITATIONS:

The limitations were self-reported dietary intake, diets assessed decades ago, possible food misclassification, possible selection bias, and residual confounding.

DISCLOSURES:

The ARIC study was funded by the US National Institutes of Health. The authors had no further disclosures.

A version of this article appeared on Medscape.com.

Publications
Publications
Topics
Article Type
Sections
Disallow All Ads
Content Gating
No Gating (article Unlocked/Free)
Alternative CME
Disqus Comments
Default
Use ProPublica
Hide sidebar & use full width
render the right sidebar.
Conference Recap Checkbox
Not Conference Recap
Clinical Edge
Display the Slideshow in this Article
Medscape Article
Display survey writer
Reuters content
Disable Inline Native ads
WebMD Article

RNA Vaccines: Risk for Heavy Menstrual Bleeding Clarified

Article Type
Changed
Thu, 02/08/2024 - 07:25

Cases of menstrual disorders, particularly unusually heavy menstrual bleeding, have been reported following RNA vaccination against COVID-19.

In France, this safety signal has been confirmed and added to the product characteristics summaries and vaccine leaflets for mRNA vaccines in October 2022. However, few studies have accurately measured this risk to date.

To address this gap in research, the French scientific interest group in the epidemiology of health products, ANSM-Cnam EPI-PHARE, conducted a study to assess the risk for heavy menstrual bleeding requiring hospitalization after COVID-19 vaccination in France.

“This study provides new evidence supporting the existence of an increased risk for heavy menstrual bleeding following COVID-19 vaccination with mRNA vaccines,” wrote the authors.
 

Study Details

The study included all women aged 15-50 years who were diagnosed with heavy menstrual bleeding in the hospital between May 12, 2021, and August 31, 2022. Participants were identified in the National Health Data System, and the study population totaled 4610 women.

Each participant was randomly matched with as many as 30 women who had not been hospitalized for abnormal genital bleeding and had similar characteristics in terms of age, department of residence, social deprivation index of the commune of residence, and contraceptive method.

Women who had a recent pregnancy, hysterectomy, or coagulation disorder within the specified time frames were excluded.

At the time of the study, 71% of cases and 70% of controls had received at least one dose of the COVID-19 vaccine. Among vaccinated participants, 68% and 66%, respectively, received a vaccination dose (first or second dose). An mRNA vaccine (Comirnaty or Spikevax) was the last vaccine for 99.8% of the population.
 

Increased Risk 

Compared with control women, those hospitalized for heavy menstrual bleeding were more likely to have received their last dose of mRNA vaccine (Comirnaty or Spikevax) in the previous 1-3 months. This association was observed for vaccination doses (odds ratio [OR], 1.20), indicating a 20% increased risk, but it was not found for booster doses (OR, 1.07).

This association was particularly notable for women residing in socially disadvantaged communities (OR, 1.28) and women not using hormonal contraception (OR, 1.28).

The risk did not appear to be increased beyond 3 months after vaccination. Researchers noted that the increased risk may have occurred earlier, considering the likely interval between initial symptoms and hospitalization.

Assuming a causal relationship, the estimated number of cases attributable to vaccination was 8 cases per million vaccinated women, totaling 103 cases among all women aged 15-50 years who were vaccinated in France between May 12, 2021, and August 31, 2022.

As of the study date and in the 3 years before the study, none of the authors had any conflicts of interest with pharmaceutical companies. 
 

This article was translated from the Medscape French edition. A version of this article appeared on Medscape.com.

Publications
Topics
Sections

Cases of menstrual disorders, particularly unusually heavy menstrual bleeding, have been reported following RNA vaccination against COVID-19.

In France, this safety signal has been confirmed and added to the product characteristics summaries and vaccine leaflets for mRNA vaccines in October 2022. However, few studies have accurately measured this risk to date.

To address this gap in research, the French scientific interest group in the epidemiology of health products, ANSM-Cnam EPI-PHARE, conducted a study to assess the risk for heavy menstrual bleeding requiring hospitalization after COVID-19 vaccination in France.

“This study provides new evidence supporting the existence of an increased risk for heavy menstrual bleeding following COVID-19 vaccination with mRNA vaccines,” wrote the authors.
 

Study Details

The study included all women aged 15-50 years who were diagnosed with heavy menstrual bleeding in the hospital between May 12, 2021, and August 31, 2022. Participants were identified in the National Health Data System, and the study population totaled 4610 women.

Each participant was randomly matched with as many as 30 women who had not been hospitalized for abnormal genital bleeding and had similar characteristics in terms of age, department of residence, social deprivation index of the commune of residence, and contraceptive method.

Women who had a recent pregnancy, hysterectomy, or coagulation disorder within the specified time frames were excluded.

At the time of the study, 71% of cases and 70% of controls had received at least one dose of the COVID-19 vaccine. Among vaccinated participants, 68% and 66%, respectively, received a vaccination dose (first or second dose). An mRNA vaccine (Comirnaty or Spikevax) was the last vaccine for 99.8% of the population.
 

Increased Risk 

Compared with control women, those hospitalized for heavy menstrual bleeding were more likely to have received their last dose of mRNA vaccine (Comirnaty or Spikevax) in the previous 1-3 months. This association was observed for vaccination doses (odds ratio [OR], 1.20), indicating a 20% increased risk, but it was not found for booster doses (OR, 1.07).

This association was particularly notable for women residing in socially disadvantaged communities (OR, 1.28) and women not using hormonal contraception (OR, 1.28).

The risk did not appear to be increased beyond 3 months after vaccination. Researchers noted that the increased risk may have occurred earlier, considering the likely interval between initial symptoms and hospitalization.

Assuming a causal relationship, the estimated number of cases attributable to vaccination was 8 cases per million vaccinated women, totaling 103 cases among all women aged 15-50 years who were vaccinated in France between May 12, 2021, and August 31, 2022.

As of the study date and in the 3 years before the study, none of the authors had any conflicts of interest with pharmaceutical companies. 
 

This article was translated from the Medscape French edition. A version of this article appeared on Medscape.com.

Cases of menstrual disorders, particularly unusually heavy menstrual bleeding, have been reported following RNA vaccination against COVID-19.

In France, this safety signal has been confirmed and added to the product characteristics summaries and vaccine leaflets for mRNA vaccines in October 2022. However, few studies have accurately measured this risk to date.

To address this gap in research, the French scientific interest group in the epidemiology of health products, ANSM-Cnam EPI-PHARE, conducted a study to assess the risk for heavy menstrual bleeding requiring hospitalization after COVID-19 vaccination in France.

“This study provides new evidence supporting the existence of an increased risk for heavy menstrual bleeding following COVID-19 vaccination with mRNA vaccines,” wrote the authors.
 

Study Details

The study included all women aged 15-50 years who were diagnosed with heavy menstrual bleeding in the hospital between May 12, 2021, and August 31, 2022. Participants were identified in the National Health Data System, and the study population totaled 4610 women.

Each participant was randomly matched with as many as 30 women who had not been hospitalized for abnormal genital bleeding and had similar characteristics in terms of age, department of residence, social deprivation index of the commune of residence, and contraceptive method.

Women who had a recent pregnancy, hysterectomy, or coagulation disorder within the specified time frames were excluded.

At the time of the study, 71% of cases and 70% of controls had received at least one dose of the COVID-19 vaccine. Among vaccinated participants, 68% and 66%, respectively, received a vaccination dose (first or second dose). An mRNA vaccine (Comirnaty or Spikevax) was the last vaccine for 99.8% of the population.
 

Increased Risk 

Compared with control women, those hospitalized for heavy menstrual bleeding were more likely to have received their last dose of mRNA vaccine (Comirnaty or Spikevax) in the previous 1-3 months. This association was observed for vaccination doses (odds ratio [OR], 1.20), indicating a 20% increased risk, but it was not found for booster doses (OR, 1.07).

This association was particularly notable for women residing in socially disadvantaged communities (OR, 1.28) and women not using hormonal contraception (OR, 1.28).

The risk did not appear to be increased beyond 3 months after vaccination. Researchers noted that the increased risk may have occurred earlier, considering the likely interval between initial symptoms and hospitalization.

Assuming a causal relationship, the estimated number of cases attributable to vaccination was 8 cases per million vaccinated women, totaling 103 cases among all women aged 15-50 years who were vaccinated in France between May 12, 2021, and August 31, 2022.

As of the study date and in the 3 years before the study, none of the authors had any conflicts of interest with pharmaceutical companies. 
 

This article was translated from the Medscape French edition. A version of this article appeared on Medscape.com.

Publications
Publications
Topics
Article Type
Sections
Disallow All Ads
Content Gating
No Gating (article Unlocked/Free)
Alternative CME
Disqus Comments
Default
Use ProPublica
Hide sidebar & use full width
render the right sidebar.
Conference Recap Checkbox
Not Conference Recap
Clinical Edge
Display the Slideshow in this Article
Medscape Article
Display survey writer
Reuters content
Disable Inline Native ads
WebMD Article

GLP-1s for Obesity: Your Questions Answered

Article Type
Changed
Wed, 02/07/2024 - 15:28

The arrival of GLP-1 receptor agonists has revolutionized treatment options for people with obesity and medical practice.

This news organization recently hosted a panel of experts across specialties — including endocrinology, gastroenterology, and obesity medicine — to discuss these potentially life-changing medications and to answer questions from the audience.

Because of the flood of queries from our audience, we asked our panelists to address some of the questions that didn’t make the recording. Their answers are below.

Beverly Tchang, MD, endocrinologist, Weill Cornell Medicine, New York City

Audience member: Can you initiate glucagon-like peptide-1 agonists (GLP-1 RAs) as a primary drug in a patient with obesity and newly diagnosed type 2 diabetes?

BT:
We often prescribe GLP-1 RAs to individuals with type 2 diabetes as a first-line medication. Guidelines from the American Diabetes Association are really emphasizing a patient-centered approach, and metformin may not be the best first-line medication anymore.

Audience member: What should clinicians know about dose titration of GLP-1 RAs in diabetic patients with renal disease, especially those in stages IV and V chronic kidney disease?

BT:
GLP-1 RAs do not need to be renally dosed, but I still recommend conferring with the patient’s nephrologist because the glomerular filtration rate might decrease in the setting of dehydration. Because GLP1s suppress the thirst, not just appetite, patients can go all day without drinking water and not feel thirsty.

Michael Camilleri, MD, gastroenterologist, Mayo Clinic, Rochester, Minnesota

Audience member: Should GLP-1 RAs be held for 1 week or 4 weeks prior to surgery to reduce the patient’s risk for aspiration? And is tapering required?

MC:
For a patient taking liraglutide, I would hold the drug for 1 week prior to surgery. For patients taking other GLP-1 RAs, including extended exenatide, I advise holding for between 2 and 3 weeks before the procedure. It’s also important to make sure the patient’s diabetes is well-controlled with other medications — not GLP-1 RAs — during this period.

After surgery, you can restart GLP-1 RA therapy once there is recovery of oral food intake and normal bowel function.

Audience member: Is treatment with GLP-1 RAs appropriate for a patient with a family history of colon cancer but an otherwise unremarkable medical and family history?

MC:
I have not seen a contraindication to receiving GLP-1 RAs based on a family history of colorectal cancer or other malignancies. An analysis of the French national healthcare insurance system database has suggested 1-3 years use of GLP-1 RAs (exenatide, liraglutide, and dulaglutide) may be linked with increased occurrence of thyroid cancer. Data from 37 randomized controlled trials and 19 real-world studies having 16,839 patients in placebo control group, 16,550 patients in active control group, and 13,330 patients in real-world studies were analyzed in a 2023 systematic review and meta-analysis. Compared to placebo or active control treatments, occurrence of pancreatic cancer, thyroid cancer, and all neoplasms — benign, malignant, and otherwise unspecified — were similar in the semaglutide group.

Toshi Iroku-Malize, MD, MPH, MBA, FAAFP, family physician, Zucker School of Medicine, Hempstead, New York

Audience member: What do you do about elevated liver functions after starting treatment with GLP-1 RAs, and what do you do when a patient has reached their weight loss goal?

TI-M:
I recommend monitoring the liver function tests, evaluating for underlying causes, such as viral hepatitis, alcohol-related damage, or problems with other medications, and consulting a gastroenterologist or liver specialist if necessary. It’s also important to discuss the risk-benefit of continuing on the GP-1 RA for that particular patient.

Audience member: What effects will GLP-1 RAs have on sleep-disordered breathing/obstructive sleep apnea (OSA)? Are you aware of any ongoing trials addressing this subject?

TI-M:
GLP-1 RAs may have beneficial effects on sleep-disordered breathing and OSA through weight loss, which can lead to a reduction in excess adipose tissue, and improvements in metabolic parameters. In terms of studies, a 2023 paper addressed this question, but more research is needed.

Audience member: Is it within a psychiatric provider’s scope of practice to prescribe GLP-1 agents for the reduction of weight gain associated with psychiatric medications?

TI-M:
Obesity medicine is an interdisciplinary process. Numerous medications prescribed for mental health can contribute to obesity, and psychiatrists can play a role in collaborating with a patient’s primary care provider and/or obesity medicine specialist to determine which medications can be adjusted or replaced. It is important to remember that obesity management is not just about medications. It requires managing nutrition and activity in addition to behavioral health issues and social determinants of health. If the clinician has had the training to manage these pillars and is comfortable managing this chronic illness — similar to diabetes, hypertension, and other conditions — then this is a possibility. Otherwise, team-based care is appropriate.

Holly Lofton, MD, obesity medicine, NYU Langone Health, New York City

Audience member: Can we safely use them on patients who have had bariatric surgery and regularly develop dumping syndrome?

HL:
These medications can be used after bariatric surgery in patients who meet the criteria for pharmacologic treatment. If a patient is having postoperative symptoms of dumping syndrome or excessive gastrointestinal losses from vomiting or diarrhea, dietary adjustments and other methods of managing the dumping syndrome in gastric bypass patients should be initiated before considering GLP-1 RAs because these patients do not have a functioning pylorus in their alimentary tract and these drugs are not indicated to treat dumping syndrome. The first-line approach typically involves reducing the patient’s intake of simple carbohydrates but can also include medications or surgical intervention when appropriate.

Audience member: Would teaching a patient to fast intermittently while they’re on GLP-1 RAs help them preserve weight loss if they choose to wean off the medication?

HL:
Personally, I feel it is best to use the titration period and the time in which the patient is actively losing weight when on GLP-1 RAs. These are the best periods to help develop an individualized treatment plan, one that includes nutrition, activity, behavior modification, and resistance training. The patient’s lifestyle plan will likely change based on their environment and other factors. Intermittent fasting can be a part of such a plan. There is no consensus as to exactly which eating pattern will help patients maintain weight once they lose the physiologic benefit of the weight loss medications. However, studies have been published that demonstrate an average weight regain of 66% or greater when patients go from taking the maximum dose of a GLP-1 RA to taking none at all. Thus, patients should still be followed closely for weight regain when they discontinue a GLP-1 RA.

A version of this article appeared on Medscape.com.

Publications
Topics
Sections

The arrival of GLP-1 receptor agonists has revolutionized treatment options for people with obesity and medical practice.

This news organization recently hosted a panel of experts across specialties — including endocrinology, gastroenterology, and obesity medicine — to discuss these potentially life-changing medications and to answer questions from the audience.

Because of the flood of queries from our audience, we asked our panelists to address some of the questions that didn’t make the recording. Their answers are below.

Beverly Tchang, MD, endocrinologist, Weill Cornell Medicine, New York City

Audience member: Can you initiate glucagon-like peptide-1 agonists (GLP-1 RAs) as a primary drug in a patient with obesity and newly diagnosed type 2 diabetes?

BT:
We often prescribe GLP-1 RAs to individuals with type 2 diabetes as a first-line medication. Guidelines from the American Diabetes Association are really emphasizing a patient-centered approach, and metformin may not be the best first-line medication anymore.

Audience member: What should clinicians know about dose titration of GLP-1 RAs in diabetic patients with renal disease, especially those in stages IV and V chronic kidney disease?

BT:
GLP-1 RAs do not need to be renally dosed, but I still recommend conferring with the patient’s nephrologist because the glomerular filtration rate might decrease in the setting of dehydration. Because GLP1s suppress the thirst, not just appetite, patients can go all day without drinking water and not feel thirsty.

Michael Camilleri, MD, gastroenterologist, Mayo Clinic, Rochester, Minnesota

Audience member: Should GLP-1 RAs be held for 1 week or 4 weeks prior to surgery to reduce the patient’s risk for aspiration? And is tapering required?

MC:
For a patient taking liraglutide, I would hold the drug for 1 week prior to surgery. For patients taking other GLP-1 RAs, including extended exenatide, I advise holding for between 2 and 3 weeks before the procedure. It’s also important to make sure the patient’s diabetes is well-controlled with other medications — not GLP-1 RAs — during this period.

After surgery, you can restart GLP-1 RA therapy once there is recovery of oral food intake and normal bowel function.

Audience member: Is treatment with GLP-1 RAs appropriate for a patient with a family history of colon cancer but an otherwise unremarkable medical and family history?

MC:
I have not seen a contraindication to receiving GLP-1 RAs based on a family history of colorectal cancer or other malignancies. An analysis of the French national healthcare insurance system database has suggested 1-3 years use of GLP-1 RAs (exenatide, liraglutide, and dulaglutide) may be linked with increased occurrence of thyroid cancer. Data from 37 randomized controlled trials and 19 real-world studies having 16,839 patients in placebo control group, 16,550 patients in active control group, and 13,330 patients in real-world studies were analyzed in a 2023 systematic review and meta-analysis. Compared to placebo or active control treatments, occurrence of pancreatic cancer, thyroid cancer, and all neoplasms — benign, malignant, and otherwise unspecified — were similar in the semaglutide group.

Toshi Iroku-Malize, MD, MPH, MBA, FAAFP, family physician, Zucker School of Medicine, Hempstead, New York

Audience member: What do you do about elevated liver functions after starting treatment with GLP-1 RAs, and what do you do when a patient has reached their weight loss goal?

TI-M:
I recommend monitoring the liver function tests, evaluating for underlying causes, such as viral hepatitis, alcohol-related damage, or problems with other medications, and consulting a gastroenterologist or liver specialist if necessary. It’s also important to discuss the risk-benefit of continuing on the GP-1 RA for that particular patient.

Audience member: What effects will GLP-1 RAs have on sleep-disordered breathing/obstructive sleep apnea (OSA)? Are you aware of any ongoing trials addressing this subject?

TI-M:
GLP-1 RAs may have beneficial effects on sleep-disordered breathing and OSA through weight loss, which can lead to a reduction in excess adipose tissue, and improvements in metabolic parameters. In terms of studies, a 2023 paper addressed this question, but more research is needed.

Audience member: Is it within a psychiatric provider’s scope of practice to prescribe GLP-1 agents for the reduction of weight gain associated with psychiatric medications?

TI-M:
Obesity medicine is an interdisciplinary process. Numerous medications prescribed for mental health can contribute to obesity, and psychiatrists can play a role in collaborating with a patient’s primary care provider and/or obesity medicine specialist to determine which medications can be adjusted or replaced. It is important to remember that obesity management is not just about medications. It requires managing nutrition and activity in addition to behavioral health issues and social determinants of health. If the clinician has had the training to manage these pillars and is comfortable managing this chronic illness — similar to diabetes, hypertension, and other conditions — then this is a possibility. Otherwise, team-based care is appropriate.

Holly Lofton, MD, obesity medicine, NYU Langone Health, New York City

Audience member: Can we safely use them on patients who have had bariatric surgery and regularly develop dumping syndrome?

HL:
These medications can be used after bariatric surgery in patients who meet the criteria for pharmacologic treatment. If a patient is having postoperative symptoms of dumping syndrome or excessive gastrointestinal losses from vomiting or diarrhea, dietary adjustments and other methods of managing the dumping syndrome in gastric bypass patients should be initiated before considering GLP-1 RAs because these patients do not have a functioning pylorus in their alimentary tract and these drugs are not indicated to treat dumping syndrome. The first-line approach typically involves reducing the patient’s intake of simple carbohydrates but can also include medications or surgical intervention when appropriate.

Audience member: Would teaching a patient to fast intermittently while they’re on GLP-1 RAs help them preserve weight loss if they choose to wean off the medication?

HL:
Personally, I feel it is best to use the titration period and the time in which the patient is actively losing weight when on GLP-1 RAs. These are the best periods to help develop an individualized treatment plan, one that includes nutrition, activity, behavior modification, and resistance training. The patient’s lifestyle plan will likely change based on their environment and other factors. Intermittent fasting can be a part of such a plan. There is no consensus as to exactly which eating pattern will help patients maintain weight once they lose the physiologic benefit of the weight loss medications. However, studies have been published that demonstrate an average weight regain of 66% or greater when patients go from taking the maximum dose of a GLP-1 RA to taking none at all. Thus, patients should still be followed closely for weight regain when they discontinue a GLP-1 RA.

A version of this article appeared on Medscape.com.

The arrival of GLP-1 receptor agonists has revolutionized treatment options for people with obesity and medical practice.

This news organization recently hosted a panel of experts across specialties — including endocrinology, gastroenterology, and obesity medicine — to discuss these potentially life-changing medications and to answer questions from the audience.

Because of the flood of queries from our audience, we asked our panelists to address some of the questions that didn’t make the recording. Their answers are below.

Beverly Tchang, MD, endocrinologist, Weill Cornell Medicine, New York City

Audience member: Can you initiate glucagon-like peptide-1 agonists (GLP-1 RAs) as a primary drug in a patient with obesity and newly diagnosed type 2 diabetes?

BT:
We often prescribe GLP-1 RAs to individuals with type 2 diabetes as a first-line medication. Guidelines from the American Diabetes Association are really emphasizing a patient-centered approach, and metformin may not be the best first-line medication anymore.

Audience member: What should clinicians know about dose titration of GLP-1 RAs in diabetic patients with renal disease, especially those in stages IV and V chronic kidney disease?

BT:
GLP-1 RAs do not need to be renally dosed, but I still recommend conferring with the patient’s nephrologist because the glomerular filtration rate might decrease in the setting of dehydration. Because GLP1s suppress the thirst, not just appetite, patients can go all day without drinking water and not feel thirsty.

Michael Camilleri, MD, gastroenterologist, Mayo Clinic, Rochester, Minnesota

Audience member: Should GLP-1 RAs be held for 1 week or 4 weeks prior to surgery to reduce the patient’s risk for aspiration? And is tapering required?

MC:
For a patient taking liraglutide, I would hold the drug for 1 week prior to surgery. For patients taking other GLP-1 RAs, including extended exenatide, I advise holding for between 2 and 3 weeks before the procedure. It’s also important to make sure the patient’s diabetes is well-controlled with other medications — not GLP-1 RAs — during this period.

After surgery, you can restart GLP-1 RA therapy once there is recovery of oral food intake and normal bowel function.

Audience member: Is treatment with GLP-1 RAs appropriate for a patient with a family history of colon cancer but an otherwise unremarkable medical and family history?

MC:
I have not seen a contraindication to receiving GLP-1 RAs based on a family history of colorectal cancer or other malignancies. An analysis of the French national healthcare insurance system database has suggested 1-3 years use of GLP-1 RAs (exenatide, liraglutide, and dulaglutide) may be linked with increased occurrence of thyroid cancer. Data from 37 randomized controlled trials and 19 real-world studies having 16,839 patients in placebo control group, 16,550 patients in active control group, and 13,330 patients in real-world studies were analyzed in a 2023 systematic review and meta-analysis. Compared to placebo or active control treatments, occurrence of pancreatic cancer, thyroid cancer, and all neoplasms — benign, malignant, and otherwise unspecified — were similar in the semaglutide group.

Toshi Iroku-Malize, MD, MPH, MBA, FAAFP, family physician, Zucker School of Medicine, Hempstead, New York

Audience member: What do you do about elevated liver functions after starting treatment with GLP-1 RAs, and what do you do when a patient has reached their weight loss goal?

TI-M:
I recommend monitoring the liver function tests, evaluating for underlying causes, such as viral hepatitis, alcohol-related damage, or problems with other medications, and consulting a gastroenterologist or liver specialist if necessary. It’s also important to discuss the risk-benefit of continuing on the GP-1 RA for that particular patient.

Audience member: What effects will GLP-1 RAs have on sleep-disordered breathing/obstructive sleep apnea (OSA)? Are you aware of any ongoing trials addressing this subject?

TI-M:
GLP-1 RAs may have beneficial effects on sleep-disordered breathing and OSA through weight loss, which can lead to a reduction in excess adipose tissue, and improvements in metabolic parameters. In terms of studies, a 2023 paper addressed this question, but more research is needed.

Audience member: Is it within a psychiatric provider’s scope of practice to prescribe GLP-1 agents for the reduction of weight gain associated with psychiatric medications?

TI-M:
Obesity medicine is an interdisciplinary process. Numerous medications prescribed for mental health can contribute to obesity, and psychiatrists can play a role in collaborating with a patient’s primary care provider and/or obesity medicine specialist to determine which medications can be adjusted or replaced. It is important to remember that obesity management is not just about medications. It requires managing nutrition and activity in addition to behavioral health issues and social determinants of health. If the clinician has had the training to manage these pillars and is comfortable managing this chronic illness — similar to diabetes, hypertension, and other conditions — then this is a possibility. Otherwise, team-based care is appropriate.

Holly Lofton, MD, obesity medicine, NYU Langone Health, New York City

Audience member: Can we safely use them on patients who have had bariatric surgery and regularly develop dumping syndrome?

HL:
These medications can be used after bariatric surgery in patients who meet the criteria for pharmacologic treatment. If a patient is having postoperative symptoms of dumping syndrome or excessive gastrointestinal losses from vomiting or diarrhea, dietary adjustments and other methods of managing the dumping syndrome in gastric bypass patients should be initiated before considering GLP-1 RAs because these patients do not have a functioning pylorus in their alimentary tract and these drugs are not indicated to treat dumping syndrome. The first-line approach typically involves reducing the patient’s intake of simple carbohydrates but can also include medications or surgical intervention when appropriate.

Audience member: Would teaching a patient to fast intermittently while they’re on GLP-1 RAs help them preserve weight loss if they choose to wean off the medication?

HL:
Personally, I feel it is best to use the titration period and the time in which the patient is actively losing weight when on GLP-1 RAs. These are the best periods to help develop an individualized treatment plan, one that includes nutrition, activity, behavior modification, and resistance training. The patient’s lifestyle plan will likely change based on their environment and other factors. Intermittent fasting can be a part of such a plan. There is no consensus as to exactly which eating pattern will help patients maintain weight once they lose the physiologic benefit of the weight loss medications. However, studies have been published that demonstrate an average weight regain of 66% or greater when patients go from taking the maximum dose of a GLP-1 RA to taking none at all. Thus, patients should still be followed closely for weight regain when they discontinue a GLP-1 RA.

A version of this article appeared on Medscape.com.

Publications
Publications
Topics
Article Type
Sections
Disallow All Ads
Content Gating
No Gating (article Unlocked/Free)
Alternative CME
Disqus Comments
Default
Use ProPublica
Hide sidebar & use full width
render the right sidebar.
Conference Recap Checkbox
Not Conference Recap
Clinical Edge
Display the Slideshow in this Article
Medscape Article
Display survey writer
Reuters content
Disable Inline Native ads
WebMD Article

SGLT2 Inhibitors Reduce Kidney Stone Risk in Type 2 Diabetes

Article Type
Changed
Wed, 02/07/2024 - 13:51

People with type 2 diabetes treated with sodium-glucose cotransporter 2 inhibitors (SGLT2) inhibitors show a significantly reduced risk of developing kidney stones compared with those treated with other commonly used diabetes drugs.

“To our knowledge, this study is the first and largest to assess the association between SGLT2 inhibitors use and risk of nephrolithiasis [kidney stones] in patients with type 2 diabetes in routine US clinical practice,” said the authors of the study, published in JAMA Internal Medicine

“Our results suggest that for patients with type 2 diabetes, the individual risk profile for developing nephrolithiasis could be a consideration when deciding which glucose-lowering agent patients should initiate,” they wrote.

The prevalence of kidney stones has been on the rise, and the problem is especially relevant to those with type 2 diabetes, which is known to have an increased risk of kidney stones, potentially causing severe pain and leading to kidney function decline.

With SGLT2 inhibitors showing renoprotective, in addition to cardiovascular benefits, first author Julie Paik, MD, MPH, an associate professor of medicine in the Division of Pharmacoepidemiology and Pharmacoeconomics and the Division of Renal (Kidney) Medicine at Brigham and Women’s Hospital in Boston, Massachusetts, and colleagues conducted an active comparator cohort study using data from three nationwide databases on patients with type 2 diabetes in routine clinical practice.

In the study’s two arms of propensity score-matched patients, 358,203 pairs of patients with type 2 diabetes were matched 1:1 to either those who were new users of SGLT2 inhibitors or glucagon-like peptide-1 (GLP-1) receptor agonists (RAs), with patients in those groups having a mean age of 61 and being about 51% female.

In addition, 331,028 pairs matched new SGLT2 inhibitor users 1:1 with didpeptidyl peptidase-4 (DPP4) inhibitor users, who also had a mean age of about 61.5 years and were about 47% female.

Over a median follow-up of 192 days, those treated with SGLT2 inhibitors had about a 31% significantly lower risk of kidney stones than GLP-1RA users (14.9 vs 21.3 events per 1000 person-years; hazard ratio [HR], 0.69).

And the SGLT2 group also had a 26% lower kidney stone risk vs DPP4 inhibitor users (14.6 vs 19.9 events per 1000 person-years; HR, 0.74).

There were no differences in the results with either groups of pairs based on sex, race, ethnicity, a history of chronic kidney disease, or obesity.

Of note, the magnitude of the risk reduction observed with SGLT2 inhibitors was greater in adults aged < 70 years than in those aged ≥ 70 years (HR, 0.85; P for interaction < .001).

The age-related difference could possibly be due to changes in stone composition that occurs with aging, which may influence SGLT2 inhibitor response, Dr Paik told this news organization.

“However, we did not have information on stone composition in our study.”

In the study, patients were taking, on average, more than two antidiabetic medications upon entrance to the study, with 13% taking thiazides and 12% taking loop diuretics. In addition, approximately half of patients discontinued SGLT2 inhibitors (52.6%) and DPP4 inhibitors (53.2%).

However, the results remained consistent after adjusting for those factors, Dr. Paik noted.
 

Mechanisms: Urinary Citrate Excretion?

Among key possible explanations for the lower risk of kidney stones with SGLT2 inhibitors is that the drugs have increased urinary citrate excretion, with one study showing a nearly 50% increase in urinary citrate excretion among patients treated with empagliflozin vs placebo over 4 weeks and other studies also showing similar increases.

“This increased urinary citrate excretion may play a pivotal role in decreasing stone risk by inhibiting supersaturation and crystallization of calcium crystals,” the authors explained.

In addition, the urinary citrate excretion could further play a role by “forming complexes with calcium and thus lowering urinary calcium concentration, and raising urinary pH, thereby reducing the risk of uric acid stones,” they added.

SGLT inhibitors’ anti-inflammatory effects could also reduce stone formation by “suppressing the expression of a stone core matrix protein, osteopontin, and markers of kidney injury, inflammation, and macrophages that promote stone formation,” the authors noted.

Ultimately, however, “while we found a lower risk of kidney stones in our study, we don’t fully understand how they lower the risk,” Dr. Paik said. The potential explanations “remain to be studied further.”

Either way, “the risk of kidney stones in a patient might be one additional consideration for a clinician to take into account when choosing among the different glucose-lowering agents for patients with type 2 diabetes,” Dr. Paik said.

The study was funded by the Division of Pharmacoepidemiology and Pharmacoeconomics, Department of Medicine, Brigham and Women’s Hospital, the National Institute of Aging the Patient-Centered Outcomes Research Institute, the US Food and Drug Administration, and the National Institute of Arthritis and Musculoskeletal and Skin Diseases. 

A version of this article appeared on Medscape.com .

Publications
Topics
Sections

People with type 2 diabetes treated with sodium-glucose cotransporter 2 inhibitors (SGLT2) inhibitors show a significantly reduced risk of developing kidney stones compared with those treated with other commonly used diabetes drugs.

“To our knowledge, this study is the first and largest to assess the association between SGLT2 inhibitors use and risk of nephrolithiasis [kidney stones] in patients with type 2 diabetes in routine US clinical practice,” said the authors of the study, published in JAMA Internal Medicine

“Our results suggest that for patients with type 2 diabetes, the individual risk profile for developing nephrolithiasis could be a consideration when deciding which glucose-lowering agent patients should initiate,” they wrote.

The prevalence of kidney stones has been on the rise, and the problem is especially relevant to those with type 2 diabetes, which is known to have an increased risk of kidney stones, potentially causing severe pain and leading to kidney function decline.

With SGLT2 inhibitors showing renoprotective, in addition to cardiovascular benefits, first author Julie Paik, MD, MPH, an associate professor of medicine in the Division of Pharmacoepidemiology and Pharmacoeconomics and the Division of Renal (Kidney) Medicine at Brigham and Women’s Hospital in Boston, Massachusetts, and colleagues conducted an active comparator cohort study using data from three nationwide databases on patients with type 2 diabetes in routine clinical practice.

In the study’s two arms of propensity score-matched patients, 358,203 pairs of patients with type 2 diabetes were matched 1:1 to either those who were new users of SGLT2 inhibitors or glucagon-like peptide-1 (GLP-1) receptor agonists (RAs), with patients in those groups having a mean age of 61 and being about 51% female.

In addition, 331,028 pairs matched new SGLT2 inhibitor users 1:1 with didpeptidyl peptidase-4 (DPP4) inhibitor users, who also had a mean age of about 61.5 years and were about 47% female.

Over a median follow-up of 192 days, those treated with SGLT2 inhibitors had about a 31% significantly lower risk of kidney stones than GLP-1RA users (14.9 vs 21.3 events per 1000 person-years; hazard ratio [HR], 0.69).

And the SGLT2 group also had a 26% lower kidney stone risk vs DPP4 inhibitor users (14.6 vs 19.9 events per 1000 person-years; HR, 0.74).

There were no differences in the results with either groups of pairs based on sex, race, ethnicity, a history of chronic kidney disease, or obesity.

Of note, the magnitude of the risk reduction observed with SGLT2 inhibitors was greater in adults aged < 70 years than in those aged ≥ 70 years (HR, 0.85; P for interaction < .001).

The age-related difference could possibly be due to changes in stone composition that occurs with aging, which may influence SGLT2 inhibitor response, Dr Paik told this news organization.

“However, we did not have information on stone composition in our study.”

In the study, patients were taking, on average, more than two antidiabetic medications upon entrance to the study, with 13% taking thiazides and 12% taking loop diuretics. In addition, approximately half of patients discontinued SGLT2 inhibitors (52.6%) and DPP4 inhibitors (53.2%).

However, the results remained consistent after adjusting for those factors, Dr. Paik noted.
 

Mechanisms: Urinary Citrate Excretion?

Among key possible explanations for the lower risk of kidney stones with SGLT2 inhibitors is that the drugs have increased urinary citrate excretion, with one study showing a nearly 50% increase in urinary citrate excretion among patients treated with empagliflozin vs placebo over 4 weeks and other studies also showing similar increases.

“This increased urinary citrate excretion may play a pivotal role in decreasing stone risk by inhibiting supersaturation and crystallization of calcium crystals,” the authors explained.

In addition, the urinary citrate excretion could further play a role by “forming complexes with calcium and thus lowering urinary calcium concentration, and raising urinary pH, thereby reducing the risk of uric acid stones,” they added.

SGLT inhibitors’ anti-inflammatory effects could also reduce stone formation by “suppressing the expression of a stone core matrix protein, osteopontin, and markers of kidney injury, inflammation, and macrophages that promote stone formation,” the authors noted.

Ultimately, however, “while we found a lower risk of kidney stones in our study, we don’t fully understand how they lower the risk,” Dr. Paik said. The potential explanations “remain to be studied further.”

Either way, “the risk of kidney stones in a patient might be one additional consideration for a clinician to take into account when choosing among the different glucose-lowering agents for patients with type 2 diabetes,” Dr. Paik said.

The study was funded by the Division of Pharmacoepidemiology and Pharmacoeconomics, Department of Medicine, Brigham and Women’s Hospital, the National Institute of Aging the Patient-Centered Outcomes Research Institute, the US Food and Drug Administration, and the National Institute of Arthritis and Musculoskeletal and Skin Diseases. 

A version of this article appeared on Medscape.com .

People with type 2 diabetes treated with sodium-glucose cotransporter 2 inhibitors (SGLT2) inhibitors show a significantly reduced risk of developing kidney stones compared with those treated with other commonly used diabetes drugs.

“To our knowledge, this study is the first and largest to assess the association between SGLT2 inhibitors use and risk of nephrolithiasis [kidney stones] in patients with type 2 diabetes in routine US clinical practice,” said the authors of the study, published in JAMA Internal Medicine

“Our results suggest that for patients with type 2 diabetes, the individual risk profile for developing nephrolithiasis could be a consideration when deciding which glucose-lowering agent patients should initiate,” they wrote.

The prevalence of kidney stones has been on the rise, and the problem is especially relevant to those with type 2 diabetes, which is known to have an increased risk of kidney stones, potentially causing severe pain and leading to kidney function decline.

With SGLT2 inhibitors showing renoprotective, in addition to cardiovascular benefits, first author Julie Paik, MD, MPH, an associate professor of medicine in the Division of Pharmacoepidemiology and Pharmacoeconomics and the Division of Renal (Kidney) Medicine at Brigham and Women’s Hospital in Boston, Massachusetts, and colleagues conducted an active comparator cohort study using data from three nationwide databases on patients with type 2 diabetes in routine clinical practice.

In the study’s two arms of propensity score-matched patients, 358,203 pairs of patients with type 2 diabetes were matched 1:1 to either those who were new users of SGLT2 inhibitors or glucagon-like peptide-1 (GLP-1) receptor agonists (RAs), with patients in those groups having a mean age of 61 and being about 51% female.

In addition, 331,028 pairs matched new SGLT2 inhibitor users 1:1 with didpeptidyl peptidase-4 (DPP4) inhibitor users, who also had a mean age of about 61.5 years and were about 47% female.

Over a median follow-up of 192 days, those treated with SGLT2 inhibitors had about a 31% significantly lower risk of kidney stones than GLP-1RA users (14.9 vs 21.3 events per 1000 person-years; hazard ratio [HR], 0.69).

And the SGLT2 group also had a 26% lower kidney stone risk vs DPP4 inhibitor users (14.6 vs 19.9 events per 1000 person-years; HR, 0.74).

There were no differences in the results with either groups of pairs based on sex, race, ethnicity, a history of chronic kidney disease, or obesity.

Of note, the magnitude of the risk reduction observed with SGLT2 inhibitors was greater in adults aged < 70 years than in those aged ≥ 70 years (HR, 0.85; P for interaction < .001).

The age-related difference could possibly be due to changes in stone composition that occurs with aging, which may influence SGLT2 inhibitor response, Dr Paik told this news organization.

“However, we did not have information on stone composition in our study.”

In the study, patients were taking, on average, more than two antidiabetic medications upon entrance to the study, with 13% taking thiazides and 12% taking loop diuretics. In addition, approximately half of patients discontinued SGLT2 inhibitors (52.6%) and DPP4 inhibitors (53.2%).

However, the results remained consistent after adjusting for those factors, Dr. Paik noted.
 

Mechanisms: Urinary Citrate Excretion?

Among key possible explanations for the lower risk of kidney stones with SGLT2 inhibitors is that the drugs have increased urinary citrate excretion, with one study showing a nearly 50% increase in urinary citrate excretion among patients treated with empagliflozin vs placebo over 4 weeks and other studies also showing similar increases.

“This increased urinary citrate excretion may play a pivotal role in decreasing stone risk by inhibiting supersaturation and crystallization of calcium crystals,” the authors explained.

In addition, the urinary citrate excretion could further play a role by “forming complexes with calcium and thus lowering urinary calcium concentration, and raising urinary pH, thereby reducing the risk of uric acid stones,” they added.

SGLT inhibitors’ anti-inflammatory effects could also reduce stone formation by “suppressing the expression of a stone core matrix protein, osteopontin, and markers of kidney injury, inflammation, and macrophages that promote stone formation,” the authors noted.

Ultimately, however, “while we found a lower risk of kidney stones in our study, we don’t fully understand how they lower the risk,” Dr. Paik said. The potential explanations “remain to be studied further.”

Either way, “the risk of kidney stones in a patient might be one additional consideration for a clinician to take into account when choosing among the different glucose-lowering agents for patients with type 2 diabetes,” Dr. Paik said.

The study was funded by the Division of Pharmacoepidemiology and Pharmacoeconomics, Department of Medicine, Brigham and Women’s Hospital, the National Institute of Aging the Patient-Centered Outcomes Research Institute, the US Food and Drug Administration, and the National Institute of Arthritis and Musculoskeletal and Skin Diseases. 

A version of this article appeared on Medscape.com .

Publications
Publications
Topics
Article Type
Sections
Article Source

FROM JAMA INTERNAL MEDICINE

Disallow All Ads
Content Gating
No Gating (article Unlocked/Free)
Alternative CME
Disqus Comments
Default
Use ProPublica
Hide sidebar & use full width
render the right sidebar.
Conference Recap Checkbox
Not Conference Recap
Clinical Edge
Display the Slideshow in this Article
Medscape Article
Display survey writer
Reuters content
Disable Inline Native ads
WebMD Article

Eli Lilly Offers Obesity Drug Directly to Consumers

Article Type
Changed
Mon, 01/08/2024 - 13:54

Eli Lilly, maker of the anti-obesity drug Zepbound, announced this week the launch of LillyDirect, a direct-to-patient portal, allowing some patients to obtain its drug for as little as $25 a month.

The move is seen as a major shift in the way these popular medications can reach patients. 

For many of the 42 million Americans with obesity, weight loss medications such as Wegovy, Saxenda, and the brand-new Zepbound can be a godsend, helping them lose the excess pounds they’ve struggled with for decades or a lifetime.

But getting these medications has been a struggle for many who are eligible. Shortages of the drugs have been one barrier, and costs of up to $1,300 monthly — the price tag without insurance coverage — are another hurdle.

But 2024 may be a much brighter year, thanks to Lilly’s new portal as well as other developments:

Insurance coverage on private health plans, while still spotty, may be improving. Federal legislators are fighting a 2003 law that forbids Medicare from paying for the medications when prescribed for obesity.

New research found that semaglutide (Wegovy) can reduce the risk of recurrent strokes and heart attacks as well as deaths from cardiovascular events in those with obesity and preexisting cardiovascular disease (or diseases of the heart and blood vessels), a finding experts said should get the attention of health insurers.

The medications, also referred to as GLP-1 agonists, work by activating the receptors of hormones (called glucagon-like peptide 1 and others) that are naturally released after eating. That, in turn, makes you feel more full, leading to weight loss of up to 22% for some. The medications are approved for those with a body mass index (BMI) of 30 or a BMI of 27 with at least one other weight-related health condition such as high blood pressure or high cholesterol. The medicines, injected weekly or more often, are prescribed along with advice about a reduced-calorie diet and increased physical activity.

LillyDirect

Eli Lilly launched its direct-to-patient portal on Thursday, providing its obesity medicine (as well as diabetes and migraine drugs) direct to the consumer. Patients can access the obesity medicines through the telehealth platform FORM. Patients reach independent telehealth providers, according to Lilly, who can complement a patient’s current doctor or be an alternative to in-patient care in some cases. 

Eli Lilly officials did not respond to requests for comment. 

Some obesity experts welcomed the new service. “Any program that improves availability and affordability of these ground-breaking medications is welcome news for our long-suffering patients,” said Louis Aronne, MD, director of the Comprehensive Weight Control Center at Weill Cornell Medicine in New York City, a long-time obesity researcher.

“It’s a great move for Lilly to do,” agreed Caroline Apovian, MD, a professor of medicine at Harvard Medical School and co-director of the Center for Weight Management and Wellness at Brigham & Women’s Hospital in Boston, who is also a veteran obesity specialist. “It is trying to help the accessibility issue and do it responsibly.” 

“The bottom line is, there is an overwhelming amount of consumer need and desire for these medications and not enough channels [to provide them],” said Zeev Neuwirth, MD, a former executive at Atrium Health who writes about health care trends. “Eli Lilly is responding to a market need that is out there and quite honestly continuing to grow.” 

There are still concerns and questions, Dr. Neuwirth said, “especially since this is to my knowledge the first of its kind in terms of a pharmaceutical manufacturer directly dispensing medication in this nontraditional way.”

He called for transparency between telehealth providers and the pharmaceutical company to rule out any conflicts of interest. 

The American College of Physicians, an organization of internal medicine doctors and others, issued a statement expressing concern. Omar T. Atiq, MD, group’s president, said his organization is “concerned by the development of websites that enable patients to order prescription medications directly from the drugmakers. While information on in-person care is available, this direct-to-consumer approach is primarily oriented around the use of telehealth services to prescribe a drug maker’s products.”

The group urged that an established patient-doctor relationship be present, or that care should happen in consultation with a doctor who does have an established relationship (the latter an option offered by Lilly). “These direct-to-consumer services have the potential to leave patients confused and misinformed about medications.”

 

 

Heart Attack, Stroke Reduction Benefits

Previous research has found that the GLP-1 medicines such as Ozempic (semaglutide), which the FDA approved to treat diabetes, also reduce the risk of cardiovascular issues such as strokes and heart attacks. Now, new research finds that semaglutide at the Wegovy dose (usually slightly higher than the Ozempic dose for diabetes) also has those benefits in those who don›t have a diabetes diagnosis but do have obesity and cardiovascular disease.

In a clinical trial sponsored by Novo Nordisk, the maker of Wegovy, half of more than 17,000 people with obesity were given semaglutide (Wegovy); the other half got a placebo. Compared to those on the placebo, those who took the Wegovy had a 20% reduction in strokes, heart attacks, and deaths from cardiovascular causes over a 33-month period. 

The study results are a “big deal,” Dr. Aronne said. The results make it clear that those with obesity but not diabetes will get the cardiovascular benefits from the treatment as well. While more analysis is necessary, he said the important point is that the study showed that reducing body weight is linked to improvement in critical health outcomes.

As the research evolves, he said, it’s going to be difficult for insurers to deny medications in the face of those findings, which promise reductions in long-term health care costs.

Insurance Coverage

In November, the American Medical Association voted to adopt a policy to urge insurance coverage for evidence-based treatment for obesity, including the new obesity medications.

“No single organization is going to be able to convince insurers and employers to cover this,” Dr. Aronne said. “But I think a prominent organization like the AMA adding their voice to the rising chorus is going to help.”

Coverage of GLP-1 medications could nearly double in 2024, according to a survey of 500 human resources decision-makers released in October by Accolade, a personalized health care advocacy and delivery company. While 25% of respondents said they currently offered coverage when the survey was done in August and September, 43% said they intend to offer coverage in 2024.

In an email, David Allen, a spokesperson for America’s Health Insurance Plans, a health care industry association, said: “Every American deserves affordable coverage and high-quality care, and that includes coverage and care for evidence-based obesity treatments and therapies.”

He said “clinical leaders and other experts at health insurance providers routinely review the evidence for all types of treatments, including treatments for obesity, and offer multiple options to patients — ranging from lifestyle changes and nutrition counseling, to surgical interventions, to prescription drugs.” 

Mr. Allen said the evidence that obesity drugs help with weight loss “is still evolving.”

“And some patients are experiencing bad effects related to these drugs such as vomiting and nausea, for example, and the likelihood of gaining the weight back when discontinuing the drugs,” he said. 

Others are fighting for Medicare coverage, while some experts contend the costs of that coverage would be overwhelming. A bipartisan bill, the Treat and Reduce Obesity Act of 2023, would allow coverage under Medicare›s prescription drug benefit for drugs used for the treatment of obesity or for weigh loss management for people who are overweight. Some say it›s an uphill climb, citing a Vanderbilt University analysis that found giving just 10% of Medicare-eligible patients the drugs would cost $13.6 billion to more than $26 billion.

However, a white paper from the University of Southern California concluded that the value to society of covering the drugs for Medicare recipients would equal nearly $1 trillion over 10 years, citing savings in hospitalizations and other health care costs.

Comprehensive insurance coverage is needed, Dr. Apovian said. Private insurance plans, Medicare, and Medicaid must all realize the importance of covering what has been now shown to be life-saving drugs, she said. 

Broader coverage might also reduce the number of patients getting obesity drugs from unreliable sources, in an effort to save money, and having adverse effects. The FDA warned against counterfeit semaglutide in December.

 

 

Long-Term Picture

Research suggests the obesity medications must be taken continuously, at least for most people, to maintain the weight loss. In a study of patients on Zepbound, Dr. Aronne and colleagues found that withdrawing the medication led people to regain weight, while continuing it led to maintaining and even increasing the initial weight loss. While some may be able to use the medications only from time to time, “the majority will have to take these on a chronic basis,” Dr. Aronne said.

Obesity, like high blood pressure and other chronic conditions, needs continuous treatment, Dr. Apovian said. No one would suggest withdrawing blood pressure medications that stabilize blood pressure; the same should be true for the obesity drugs, she said.

Dr. Apovian consults for FORM, the telehealth platform Lilly uses for LillyDirect, and consults for Novo Nordisk, which makes Saxenda and Wegovy. Dr. Aronne is a consultant and investigator for Novo Nordisk, Eli Lilly, and other companies.

A version of this article appeared on WebMD.com.

Publications
Topics
Sections

Eli Lilly, maker of the anti-obesity drug Zepbound, announced this week the launch of LillyDirect, a direct-to-patient portal, allowing some patients to obtain its drug for as little as $25 a month.

The move is seen as a major shift in the way these popular medications can reach patients. 

For many of the 42 million Americans with obesity, weight loss medications such as Wegovy, Saxenda, and the brand-new Zepbound can be a godsend, helping them lose the excess pounds they’ve struggled with for decades or a lifetime.

But getting these medications has been a struggle for many who are eligible. Shortages of the drugs have been one barrier, and costs of up to $1,300 monthly — the price tag without insurance coverage — are another hurdle.

But 2024 may be a much brighter year, thanks to Lilly’s new portal as well as other developments:

Insurance coverage on private health plans, while still spotty, may be improving. Federal legislators are fighting a 2003 law that forbids Medicare from paying for the medications when prescribed for obesity.

New research found that semaglutide (Wegovy) can reduce the risk of recurrent strokes and heart attacks as well as deaths from cardiovascular events in those with obesity and preexisting cardiovascular disease (or diseases of the heart and blood vessels), a finding experts said should get the attention of health insurers.

The medications, also referred to as GLP-1 agonists, work by activating the receptors of hormones (called glucagon-like peptide 1 and others) that are naturally released after eating. That, in turn, makes you feel more full, leading to weight loss of up to 22% for some. The medications are approved for those with a body mass index (BMI) of 30 or a BMI of 27 with at least one other weight-related health condition such as high blood pressure or high cholesterol. The medicines, injected weekly or more often, are prescribed along with advice about a reduced-calorie diet and increased physical activity.

LillyDirect

Eli Lilly launched its direct-to-patient portal on Thursday, providing its obesity medicine (as well as diabetes and migraine drugs) direct to the consumer. Patients can access the obesity medicines through the telehealth platform FORM. Patients reach independent telehealth providers, according to Lilly, who can complement a patient’s current doctor or be an alternative to in-patient care in some cases. 

Eli Lilly officials did not respond to requests for comment. 

Some obesity experts welcomed the new service. “Any program that improves availability and affordability of these ground-breaking medications is welcome news for our long-suffering patients,” said Louis Aronne, MD, director of the Comprehensive Weight Control Center at Weill Cornell Medicine in New York City, a long-time obesity researcher.

“It’s a great move for Lilly to do,” agreed Caroline Apovian, MD, a professor of medicine at Harvard Medical School and co-director of the Center for Weight Management and Wellness at Brigham & Women’s Hospital in Boston, who is also a veteran obesity specialist. “It is trying to help the accessibility issue and do it responsibly.” 

“The bottom line is, there is an overwhelming amount of consumer need and desire for these medications and not enough channels [to provide them],” said Zeev Neuwirth, MD, a former executive at Atrium Health who writes about health care trends. “Eli Lilly is responding to a market need that is out there and quite honestly continuing to grow.” 

There are still concerns and questions, Dr. Neuwirth said, “especially since this is to my knowledge the first of its kind in terms of a pharmaceutical manufacturer directly dispensing medication in this nontraditional way.”

He called for transparency between telehealth providers and the pharmaceutical company to rule out any conflicts of interest. 

The American College of Physicians, an organization of internal medicine doctors and others, issued a statement expressing concern. Omar T. Atiq, MD, group’s president, said his organization is “concerned by the development of websites that enable patients to order prescription medications directly from the drugmakers. While information on in-person care is available, this direct-to-consumer approach is primarily oriented around the use of telehealth services to prescribe a drug maker’s products.”

The group urged that an established patient-doctor relationship be present, or that care should happen in consultation with a doctor who does have an established relationship (the latter an option offered by Lilly). “These direct-to-consumer services have the potential to leave patients confused and misinformed about medications.”

 

 

Heart Attack, Stroke Reduction Benefits

Previous research has found that the GLP-1 medicines such as Ozempic (semaglutide), which the FDA approved to treat diabetes, also reduce the risk of cardiovascular issues such as strokes and heart attacks. Now, new research finds that semaglutide at the Wegovy dose (usually slightly higher than the Ozempic dose for diabetes) also has those benefits in those who don›t have a diabetes diagnosis but do have obesity and cardiovascular disease.

In a clinical trial sponsored by Novo Nordisk, the maker of Wegovy, half of more than 17,000 people with obesity were given semaglutide (Wegovy); the other half got a placebo. Compared to those on the placebo, those who took the Wegovy had a 20% reduction in strokes, heart attacks, and deaths from cardiovascular causes over a 33-month period. 

The study results are a “big deal,” Dr. Aronne said. The results make it clear that those with obesity but not diabetes will get the cardiovascular benefits from the treatment as well. While more analysis is necessary, he said the important point is that the study showed that reducing body weight is linked to improvement in critical health outcomes.

As the research evolves, he said, it’s going to be difficult for insurers to deny medications in the face of those findings, which promise reductions in long-term health care costs.

Insurance Coverage

In November, the American Medical Association voted to adopt a policy to urge insurance coverage for evidence-based treatment for obesity, including the new obesity medications.

“No single organization is going to be able to convince insurers and employers to cover this,” Dr. Aronne said. “But I think a prominent organization like the AMA adding their voice to the rising chorus is going to help.”

Coverage of GLP-1 medications could nearly double in 2024, according to a survey of 500 human resources decision-makers released in October by Accolade, a personalized health care advocacy and delivery company. While 25% of respondents said they currently offered coverage when the survey was done in August and September, 43% said they intend to offer coverage in 2024.

In an email, David Allen, a spokesperson for America’s Health Insurance Plans, a health care industry association, said: “Every American deserves affordable coverage and high-quality care, and that includes coverage and care for evidence-based obesity treatments and therapies.”

He said “clinical leaders and other experts at health insurance providers routinely review the evidence for all types of treatments, including treatments for obesity, and offer multiple options to patients — ranging from lifestyle changes and nutrition counseling, to surgical interventions, to prescription drugs.” 

Mr. Allen said the evidence that obesity drugs help with weight loss “is still evolving.”

“And some patients are experiencing bad effects related to these drugs such as vomiting and nausea, for example, and the likelihood of gaining the weight back when discontinuing the drugs,” he said. 

Others are fighting for Medicare coverage, while some experts contend the costs of that coverage would be overwhelming. A bipartisan bill, the Treat and Reduce Obesity Act of 2023, would allow coverage under Medicare›s prescription drug benefit for drugs used for the treatment of obesity or for weigh loss management for people who are overweight. Some say it›s an uphill climb, citing a Vanderbilt University analysis that found giving just 10% of Medicare-eligible patients the drugs would cost $13.6 billion to more than $26 billion.

However, a white paper from the University of Southern California concluded that the value to society of covering the drugs for Medicare recipients would equal nearly $1 trillion over 10 years, citing savings in hospitalizations and other health care costs.

Comprehensive insurance coverage is needed, Dr. Apovian said. Private insurance plans, Medicare, and Medicaid must all realize the importance of covering what has been now shown to be life-saving drugs, she said. 

Broader coverage might also reduce the number of patients getting obesity drugs from unreliable sources, in an effort to save money, and having adverse effects. The FDA warned against counterfeit semaglutide in December.

 

 

Long-Term Picture

Research suggests the obesity medications must be taken continuously, at least for most people, to maintain the weight loss. In a study of patients on Zepbound, Dr. Aronne and colleagues found that withdrawing the medication led people to regain weight, while continuing it led to maintaining and even increasing the initial weight loss. While some may be able to use the medications only from time to time, “the majority will have to take these on a chronic basis,” Dr. Aronne said.

Obesity, like high blood pressure and other chronic conditions, needs continuous treatment, Dr. Apovian said. No one would suggest withdrawing blood pressure medications that stabilize blood pressure; the same should be true for the obesity drugs, she said.

Dr. Apovian consults for FORM, the telehealth platform Lilly uses for LillyDirect, and consults for Novo Nordisk, which makes Saxenda and Wegovy. Dr. Aronne is a consultant and investigator for Novo Nordisk, Eli Lilly, and other companies.

A version of this article appeared on WebMD.com.

Eli Lilly, maker of the anti-obesity drug Zepbound, announced this week the launch of LillyDirect, a direct-to-patient portal, allowing some patients to obtain its drug for as little as $25 a month.

The move is seen as a major shift in the way these popular medications can reach patients. 

For many of the 42 million Americans with obesity, weight loss medications such as Wegovy, Saxenda, and the brand-new Zepbound can be a godsend, helping them lose the excess pounds they’ve struggled with for decades or a lifetime.

But getting these medications has been a struggle for many who are eligible. Shortages of the drugs have been one barrier, and costs of up to $1,300 monthly — the price tag without insurance coverage — are another hurdle.

But 2024 may be a much brighter year, thanks to Lilly’s new portal as well as other developments:

Insurance coverage on private health plans, while still spotty, may be improving. Federal legislators are fighting a 2003 law that forbids Medicare from paying for the medications when prescribed for obesity.

New research found that semaglutide (Wegovy) can reduce the risk of recurrent strokes and heart attacks as well as deaths from cardiovascular events in those with obesity and preexisting cardiovascular disease (or diseases of the heart and blood vessels), a finding experts said should get the attention of health insurers.

The medications, also referred to as GLP-1 agonists, work by activating the receptors of hormones (called glucagon-like peptide 1 and others) that are naturally released after eating. That, in turn, makes you feel more full, leading to weight loss of up to 22% for some. The medications are approved for those with a body mass index (BMI) of 30 or a BMI of 27 with at least one other weight-related health condition such as high blood pressure or high cholesterol. The medicines, injected weekly or more often, are prescribed along with advice about a reduced-calorie diet and increased physical activity.

LillyDirect

Eli Lilly launched its direct-to-patient portal on Thursday, providing its obesity medicine (as well as diabetes and migraine drugs) direct to the consumer. Patients can access the obesity medicines through the telehealth platform FORM. Patients reach independent telehealth providers, according to Lilly, who can complement a patient’s current doctor or be an alternative to in-patient care in some cases. 

Eli Lilly officials did not respond to requests for comment. 

Some obesity experts welcomed the new service. “Any program that improves availability and affordability of these ground-breaking medications is welcome news for our long-suffering patients,” said Louis Aronne, MD, director of the Comprehensive Weight Control Center at Weill Cornell Medicine in New York City, a long-time obesity researcher.

“It’s a great move for Lilly to do,” agreed Caroline Apovian, MD, a professor of medicine at Harvard Medical School and co-director of the Center for Weight Management and Wellness at Brigham & Women’s Hospital in Boston, who is also a veteran obesity specialist. “It is trying to help the accessibility issue and do it responsibly.” 

“The bottom line is, there is an overwhelming amount of consumer need and desire for these medications and not enough channels [to provide them],” said Zeev Neuwirth, MD, a former executive at Atrium Health who writes about health care trends. “Eli Lilly is responding to a market need that is out there and quite honestly continuing to grow.” 

There are still concerns and questions, Dr. Neuwirth said, “especially since this is to my knowledge the first of its kind in terms of a pharmaceutical manufacturer directly dispensing medication in this nontraditional way.”

He called for transparency between telehealth providers and the pharmaceutical company to rule out any conflicts of interest. 

The American College of Physicians, an organization of internal medicine doctors and others, issued a statement expressing concern. Omar T. Atiq, MD, group’s president, said his organization is “concerned by the development of websites that enable patients to order prescription medications directly from the drugmakers. While information on in-person care is available, this direct-to-consumer approach is primarily oriented around the use of telehealth services to prescribe a drug maker’s products.”

The group urged that an established patient-doctor relationship be present, or that care should happen in consultation with a doctor who does have an established relationship (the latter an option offered by Lilly). “These direct-to-consumer services have the potential to leave patients confused and misinformed about medications.”

 

 

Heart Attack, Stroke Reduction Benefits

Previous research has found that the GLP-1 medicines such as Ozempic (semaglutide), which the FDA approved to treat diabetes, also reduce the risk of cardiovascular issues such as strokes and heart attacks. Now, new research finds that semaglutide at the Wegovy dose (usually slightly higher than the Ozempic dose for diabetes) also has those benefits in those who don›t have a diabetes diagnosis but do have obesity and cardiovascular disease.

In a clinical trial sponsored by Novo Nordisk, the maker of Wegovy, half of more than 17,000 people with obesity were given semaglutide (Wegovy); the other half got a placebo. Compared to those on the placebo, those who took the Wegovy had a 20% reduction in strokes, heart attacks, and deaths from cardiovascular causes over a 33-month period. 

The study results are a “big deal,” Dr. Aronne said. The results make it clear that those with obesity but not diabetes will get the cardiovascular benefits from the treatment as well. While more analysis is necessary, he said the important point is that the study showed that reducing body weight is linked to improvement in critical health outcomes.

As the research evolves, he said, it’s going to be difficult for insurers to deny medications in the face of those findings, which promise reductions in long-term health care costs.

Insurance Coverage

In November, the American Medical Association voted to adopt a policy to urge insurance coverage for evidence-based treatment for obesity, including the new obesity medications.

“No single organization is going to be able to convince insurers and employers to cover this,” Dr. Aronne said. “But I think a prominent organization like the AMA adding their voice to the rising chorus is going to help.”

Coverage of GLP-1 medications could nearly double in 2024, according to a survey of 500 human resources decision-makers released in October by Accolade, a personalized health care advocacy and delivery company. While 25% of respondents said they currently offered coverage when the survey was done in August and September, 43% said they intend to offer coverage in 2024.

In an email, David Allen, a spokesperson for America’s Health Insurance Plans, a health care industry association, said: “Every American deserves affordable coverage and high-quality care, and that includes coverage and care for evidence-based obesity treatments and therapies.”

He said “clinical leaders and other experts at health insurance providers routinely review the evidence for all types of treatments, including treatments for obesity, and offer multiple options to patients — ranging from lifestyle changes and nutrition counseling, to surgical interventions, to prescription drugs.” 

Mr. Allen said the evidence that obesity drugs help with weight loss “is still evolving.”

“And some patients are experiencing bad effects related to these drugs such as vomiting and nausea, for example, and the likelihood of gaining the weight back when discontinuing the drugs,” he said. 

Others are fighting for Medicare coverage, while some experts contend the costs of that coverage would be overwhelming. A bipartisan bill, the Treat and Reduce Obesity Act of 2023, would allow coverage under Medicare›s prescription drug benefit for drugs used for the treatment of obesity or for weigh loss management for people who are overweight. Some say it›s an uphill climb, citing a Vanderbilt University analysis that found giving just 10% of Medicare-eligible patients the drugs would cost $13.6 billion to more than $26 billion.

However, a white paper from the University of Southern California concluded that the value to society of covering the drugs for Medicare recipients would equal nearly $1 trillion over 10 years, citing savings in hospitalizations and other health care costs.

Comprehensive insurance coverage is needed, Dr. Apovian said. Private insurance plans, Medicare, and Medicaid must all realize the importance of covering what has been now shown to be life-saving drugs, she said. 

Broader coverage might also reduce the number of patients getting obesity drugs from unreliable sources, in an effort to save money, and having adverse effects. The FDA warned against counterfeit semaglutide in December.

 

 

Long-Term Picture

Research suggests the obesity medications must be taken continuously, at least for most people, to maintain the weight loss. In a study of patients on Zepbound, Dr. Aronne and colleagues found that withdrawing the medication led people to regain weight, while continuing it led to maintaining and even increasing the initial weight loss. While some may be able to use the medications only from time to time, “the majority will have to take these on a chronic basis,” Dr. Aronne said.

Obesity, like high blood pressure and other chronic conditions, needs continuous treatment, Dr. Apovian said. No one would suggest withdrawing blood pressure medications that stabilize blood pressure; the same should be true for the obesity drugs, she said.

Dr. Apovian consults for FORM, the telehealth platform Lilly uses for LillyDirect, and consults for Novo Nordisk, which makes Saxenda and Wegovy. Dr. Aronne is a consultant and investigator for Novo Nordisk, Eli Lilly, and other companies.

A version of this article appeared on WebMD.com.

Publications
Publications
Topics
Article Type
Sections
Disallow All Ads
Content Gating
No Gating (article Unlocked/Free)
Alternative CME
Disqus Comments
Default
Use ProPublica
Hide sidebar & use full width
render the right sidebar.
Conference Recap Checkbox
Not Conference Recap
Clinical Edge
Display the Slideshow in this Article
Medscape Article
Display survey writer
Reuters content
Disable Inline Native ads
WebMD Article

Long COVID Has Caused Thousands of US Deaths: New CDC Data

Article Type
Changed
Thu, 01/04/2024 - 12:05

While COVID has now claimed more than 1 million lives in the United States alone, these aren’t the only fatalities caused at least in part by the virus. A small but growing number of Americans are surviving acute infections only to succumb months later to the lingering health problems caused by long COVID.

Much of the attention on long COVID has centered on the sometimes debilitating symptoms that strike people with the condition, with no formal diagnostic tests or standard treatments available, and the effect it has on quality of life. But new figures from the US Centers for Disease Control and Prevention (CDC) show that long COVID can also be deadly.

More than 5000 Americans have died from long COVID since the start of the pandemic, according to new estimates from the CDC.

This total, based on death certificate data collected by the CDC, includes a preliminary tally of 1491 long COVID deaths in 2023 in addition to 3544 fatalities previously reported from January 2020 through June 2022.

Guidance issued in 2023 on how to formally report long COVID as a cause of death on death certificates should help get a more accurate count of these fatalities going forward, said Robert Anderson, PhD, chief mortality statistician for the CDC, Atlanta, Georgia.

“We hope that the guidance will help cause of death certifiers be more aware of the impact of long COVID and more likely to report long COVID as a cause of death when appropriate,” Dr. Anderson said. “That said, we do not expect that this guidance will have a dramatic impact on the trend.”

There’s no standard definition or diagnostic test for long COVID. It’s typically diagnosed when people have symptoms at least 3 months after an acute infection that weren’t present before they got sick. As of the end of last year, about 7% of American adults had experienced long COVID at some point, the CDC estimated in September 2023.

The new death tally indicates long COVID remains a significant public health threat and is likely to grow in the years ahead, even though the pandemic may no longer be considered a global health crisis, experts said.

For example, the death certificate figures indicate:

COVID-19 was the third leading cause of American deaths in 2020 and 2021, and the fourth leading cause of death in the United States in 2023.

Nearly 1% of the more than one million deaths related to COVID-19 since the start of the pandemic have been attributed to long COVID, according to data released by the CDC.

The proportion of COVID-related deaths from long COVID peaked in June 2021 at 1.2% and again in April 2022 at 3.8%, according to the CDC. Both of these peaks coincided with periods of declining fatalities from acute infections.

“I do expect that deaths associated with long COVID will make up an increasingly larger proportion of total deaths associated with COVID-19,” said Mark Czeisler, PhD, a researcher at Harvard Medical School, Boston, Massachusetts, who has studied long COVID fatalities. 

Months and even years after an acute infection, long COVID can contribute to serious and potentially life-threatening conditions that impact nearly every major system in the body, according to the CDC guidelines for identifying the condition on death certificates. 

This means long COVID may often be listed as an underlying cause of death when people with this condition die of issues related to their heart, lungs, brain or kidneys, the CDC guidelines noted.

The risk for long COVID fatalities remains elevated for at least 6 months for people with milder acute infections and for at least 2 years in severe cases that require hospitalization, some previous research suggested.

As happens with other acute infections, certain people are more at risk for fatal case of long COVID. Age, race, and ethnicity have all been cited as risk factors by researchers who have been tracking the condition since the start of the pandemic.

Half of long COVID fatalities from July 2021 to June 2022 occurred in people aged 65 years and older, and another 23% were recorded among people aged 50-64 years old, according a report from CDC.

Long COVID death rates also varied by race and ethnicity, from a high of 14.1 cases per million among America Indian and Alaskan natives to a low of 1.5 cases per million among Asian people, the CDC found. Death rates per million were 6.7 for White individuals, 6.4 for Black people, and 4.7 for Hispanic people.

The disproportionate share of Black and Hispanic people who developed and died from severe acute infections may have left fewer survivors to develop long COVID, limiting long COVID fatalities among these groups, the CDC report concluded.

It’s also possible that long COVID fatalities were undercounted in these populations because they faced challenges accessing healthcare or seeing providers who could recognize the hallmark symptoms of long COVID.

It’s also difficult to distinguish between how many deaths related to the virus ultimately occur as a result of long COVID rather than acute infections. That’s because it may depend on a variety of factors, including how consistently medical examiners follow the CDC guidelines, said Ziyad Al-Aly, MD, chief of research at the Veterans Affairs, St. Louis Health Care System and a senior clinical epidemiologist at Washington University in St. Louis.

“Long COVID remains massively underdiagnosed, and death in people with long COVID is misattributed to other things,” Dr. Al-Aly said.

An accurate test for long COVID could help lead to a more accurate count of these fatalities, Dr. Czeisler said. Some preliminary research suggests that it might one day be possible to diagnose long COVID with a blood test.

“The timeline for such a test and the extent to which it would be widely applied is uncertain,” Dr. Czeisler noted, “though that would certainly be a gamechanger.”

A version of this article appeared on Medscape.com.

Publications
Topics
Sections

While COVID has now claimed more than 1 million lives in the United States alone, these aren’t the only fatalities caused at least in part by the virus. A small but growing number of Americans are surviving acute infections only to succumb months later to the lingering health problems caused by long COVID.

Much of the attention on long COVID has centered on the sometimes debilitating symptoms that strike people with the condition, with no formal diagnostic tests or standard treatments available, and the effect it has on quality of life. But new figures from the US Centers for Disease Control and Prevention (CDC) show that long COVID can also be deadly.

More than 5000 Americans have died from long COVID since the start of the pandemic, according to new estimates from the CDC.

This total, based on death certificate data collected by the CDC, includes a preliminary tally of 1491 long COVID deaths in 2023 in addition to 3544 fatalities previously reported from January 2020 through June 2022.

Guidance issued in 2023 on how to formally report long COVID as a cause of death on death certificates should help get a more accurate count of these fatalities going forward, said Robert Anderson, PhD, chief mortality statistician for the CDC, Atlanta, Georgia.

“We hope that the guidance will help cause of death certifiers be more aware of the impact of long COVID and more likely to report long COVID as a cause of death when appropriate,” Dr. Anderson said. “That said, we do not expect that this guidance will have a dramatic impact on the trend.”

There’s no standard definition or diagnostic test for long COVID. It’s typically diagnosed when people have symptoms at least 3 months after an acute infection that weren’t present before they got sick. As of the end of last year, about 7% of American adults had experienced long COVID at some point, the CDC estimated in September 2023.

The new death tally indicates long COVID remains a significant public health threat and is likely to grow in the years ahead, even though the pandemic may no longer be considered a global health crisis, experts said.

For example, the death certificate figures indicate:

COVID-19 was the third leading cause of American deaths in 2020 and 2021, and the fourth leading cause of death in the United States in 2023.

Nearly 1% of the more than one million deaths related to COVID-19 since the start of the pandemic have been attributed to long COVID, according to data released by the CDC.

The proportion of COVID-related deaths from long COVID peaked in June 2021 at 1.2% and again in April 2022 at 3.8%, according to the CDC. Both of these peaks coincided with periods of declining fatalities from acute infections.

“I do expect that deaths associated with long COVID will make up an increasingly larger proportion of total deaths associated with COVID-19,” said Mark Czeisler, PhD, a researcher at Harvard Medical School, Boston, Massachusetts, who has studied long COVID fatalities. 

Months and even years after an acute infection, long COVID can contribute to serious and potentially life-threatening conditions that impact nearly every major system in the body, according to the CDC guidelines for identifying the condition on death certificates. 

This means long COVID may often be listed as an underlying cause of death when people with this condition die of issues related to their heart, lungs, brain or kidneys, the CDC guidelines noted.

The risk for long COVID fatalities remains elevated for at least 6 months for people with milder acute infections and for at least 2 years in severe cases that require hospitalization, some previous research suggested.

As happens with other acute infections, certain people are more at risk for fatal case of long COVID. Age, race, and ethnicity have all been cited as risk factors by researchers who have been tracking the condition since the start of the pandemic.

Half of long COVID fatalities from July 2021 to June 2022 occurred in people aged 65 years and older, and another 23% were recorded among people aged 50-64 years old, according a report from CDC.

Long COVID death rates also varied by race and ethnicity, from a high of 14.1 cases per million among America Indian and Alaskan natives to a low of 1.5 cases per million among Asian people, the CDC found. Death rates per million were 6.7 for White individuals, 6.4 for Black people, and 4.7 for Hispanic people.

The disproportionate share of Black and Hispanic people who developed and died from severe acute infections may have left fewer survivors to develop long COVID, limiting long COVID fatalities among these groups, the CDC report concluded.

It’s also possible that long COVID fatalities were undercounted in these populations because they faced challenges accessing healthcare or seeing providers who could recognize the hallmark symptoms of long COVID.

It’s also difficult to distinguish between how many deaths related to the virus ultimately occur as a result of long COVID rather than acute infections. That’s because it may depend on a variety of factors, including how consistently medical examiners follow the CDC guidelines, said Ziyad Al-Aly, MD, chief of research at the Veterans Affairs, St. Louis Health Care System and a senior clinical epidemiologist at Washington University in St. Louis.

“Long COVID remains massively underdiagnosed, and death in people with long COVID is misattributed to other things,” Dr. Al-Aly said.

An accurate test for long COVID could help lead to a more accurate count of these fatalities, Dr. Czeisler said. Some preliminary research suggests that it might one day be possible to diagnose long COVID with a blood test.

“The timeline for such a test and the extent to which it would be widely applied is uncertain,” Dr. Czeisler noted, “though that would certainly be a gamechanger.”

A version of this article appeared on Medscape.com.

While COVID has now claimed more than 1 million lives in the United States alone, these aren’t the only fatalities caused at least in part by the virus. A small but growing number of Americans are surviving acute infections only to succumb months later to the lingering health problems caused by long COVID.

Much of the attention on long COVID has centered on the sometimes debilitating symptoms that strike people with the condition, with no formal diagnostic tests or standard treatments available, and the effect it has on quality of life. But new figures from the US Centers for Disease Control and Prevention (CDC) show that long COVID can also be deadly.

More than 5000 Americans have died from long COVID since the start of the pandemic, according to new estimates from the CDC.

This total, based on death certificate data collected by the CDC, includes a preliminary tally of 1491 long COVID deaths in 2023 in addition to 3544 fatalities previously reported from January 2020 through June 2022.

Guidance issued in 2023 on how to formally report long COVID as a cause of death on death certificates should help get a more accurate count of these fatalities going forward, said Robert Anderson, PhD, chief mortality statistician for the CDC, Atlanta, Georgia.

“We hope that the guidance will help cause of death certifiers be more aware of the impact of long COVID and more likely to report long COVID as a cause of death when appropriate,” Dr. Anderson said. “That said, we do not expect that this guidance will have a dramatic impact on the trend.”

There’s no standard definition or diagnostic test for long COVID. It’s typically diagnosed when people have symptoms at least 3 months after an acute infection that weren’t present before they got sick. As of the end of last year, about 7% of American adults had experienced long COVID at some point, the CDC estimated in September 2023.

The new death tally indicates long COVID remains a significant public health threat and is likely to grow in the years ahead, even though the pandemic may no longer be considered a global health crisis, experts said.

For example, the death certificate figures indicate:

COVID-19 was the third leading cause of American deaths in 2020 and 2021, and the fourth leading cause of death in the United States in 2023.

Nearly 1% of the more than one million deaths related to COVID-19 since the start of the pandemic have been attributed to long COVID, according to data released by the CDC.

The proportion of COVID-related deaths from long COVID peaked in June 2021 at 1.2% and again in April 2022 at 3.8%, according to the CDC. Both of these peaks coincided with periods of declining fatalities from acute infections.

“I do expect that deaths associated with long COVID will make up an increasingly larger proportion of total deaths associated with COVID-19,” said Mark Czeisler, PhD, a researcher at Harvard Medical School, Boston, Massachusetts, who has studied long COVID fatalities. 

Months and even years after an acute infection, long COVID can contribute to serious and potentially life-threatening conditions that impact nearly every major system in the body, according to the CDC guidelines for identifying the condition on death certificates. 

This means long COVID may often be listed as an underlying cause of death when people with this condition die of issues related to their heart, lungs, brain or kidneys, the CDC guidelines noted.

The risk for long COVID fatalities remains elevated for at least 6 months for people with milder acute infections and for at least 2 years in severe cases that require hospitalization, some previous research suggested.

As happens with other acute infections, certain people are more at risk for fatal case of long COVID. Age, race, and ethnicity have all been cited as risk factors by researchers who have been tracking the condition since the start of the pandemic.

Half of long COVID fatalities from July 2021 to June 2022 occurred in people aged 65 years and older, and another 23% were recorded among people aged 50-64 years old, according a report from CDC.

Long COVID death rates also varied by race and ethnicity, from a high of 14.1 cases per million among America Indian and Alaskan natives to a low of 1.5 cases per million among Asian people, the CDC found. Death rates per million were 6.7 for White individuals, 6.4 for Black people, and 4.7 for Hispanic people.

The disproportionate share of Black and Hispanic people who developed and died from severe acute infections may have left fewer survivors to develop long COVID, limiting long COVID fatalities among these groups, the CDC report concluded.

It’s also possible that long COVID fatalities were undercounted in these populations because they faced challenges accessing healthcare or seeing providers who could recognize the hallmark symptoms of long COVID.

It’s also difficult to distinguish between how many deaths related to the virus ultimately occur as a result of long COVID rather than acute infections. That’s because it may depend on a variety of factors, including how consistently medical examiners follow the CDC guidelines, said Ziyad Al-Aly, MD, chief of research at the Veterans Affairs, St. Louis Health Care System and a senior clinical epidemiologist at Washington University in St. Louis.

“Long COVID remains massively underdiagnosed, and death in people with long COVID is misattributed to other things,” Dr. Al-Aly said.

An accurate test for long COVID could help lead to a more accurate count of these fatalities, Dr. Czeisler said. Some preliminary research suggests that it might one day be possible to diagnose long COVID with a blood test.

“The timeline for such a test and the extent to which it would be widely applied is uncertain,” Dr. Czeisler noted, “though that would certainly be a gamechanger.”

A version of this article appeared on Medscape.com.

Publications
Publications
Topics
Article Type
Sections
Disallow All Ads
Content Gating
No Gating (article Unlocked/Free)
Alternative CME
Disqus Comments
Default
Use ProPublica
Hide sidebar & use full width
render the right sidebar.
Conference Recap Checkbox
Not Conference Recap
Clinical Edge
Display the Slideshow in this Article
Medscape Article
Display survey writer
Reuters content
Disable Inline Native ads
WebMD Article

JAMA Internal Medicine Editor Recaps 2023’s High-Impact Research

Article Type
Changed
Tue, 01/02/2024 - 16:08

Harvard Medical School’s Sharon K. Inouye, MD, MPH, is editor in chief of JAMA Internal Medicine and a leading voice in American gerontology. We asked her to choose five of the influential journal’s most impactful studies from 2023 and highlight important take-home messages for internists and their colleagues.
 

Q: One of the studies you chose suggests that the antiviral nirmatrelvir (Paxlovid) can ward off long COVID. Could you recap the findings?

A: Researchers followed a group of more than 280,000 Department of Veterans Affairs patients who were seen in 2022, had a positive COVID test, and had at least one risk factor for severe COVID. They focused on those who survived to 30 days after their COVID infection and compared those who received the drug within the first 5 days of a positive test with an equivalent control group.

They found that 13 long COVID symptoms were all significantly less common (relative risk = 0.74) in those who received nirmatrelvir. This was true no matter whether they’d ever had a COVID vaccination.
 

Q: How should this research affect clinical practice?

A: You can’t generalize from this to everyone because, of course, not everyone was included in this study. But it is highly suggestive that this drug is very effective for preventing long COVID.

Nirmatrelvir was touted as being able to shorten duration of illness and prevent hospitalization. But if you were low risk or you were already well into your COVID course, it wasn’t like rush, rush, rush to the doctor to get it.

This changes that equation because we know long COVID is such a huge issue. The vast majority of doctors who work with COVID patients and know this are now being more aggressive about prescribing it.
 

Q: What about patients whom the CDC considers to be at less risk — people with up-to-date vaccinations who are under 50 with mild-to-moderate COVID and no higher-risk medical conditions? Should they take nirmatrelvir?

A: The evidence is not 100% in yet. A study like this one needs to be repeated and include younger people without any risk factors to see if we see the same thing. So it’s a personal choice, and a personal calculus needs to be done. A lot of people are making that choice [to take the drug], and it can be a rational decision.

Q: You also chose a study that links high thyroid hormone levels to higher rates of dementia. What did it reveal?

A: This study looks at patients who had thyrotoxicosis — a thyroid level that’s too high — from hormone produced endogenously, and exogenously. Researchers tracked almost 66,000 patients aged 65 and older and found that thyrotoxicosis from all causes, whether it was endogenous or exogenous, was linked to an increased risk of dementia in a dose-response relationship (adjusted hazard ratio = 1.39).

Q: Is there a clinical take-home message here?

A: When we start patients on thyroid medication, they don’t always get reassessed on a regular basis. Given this finding, a TSH [thyroid-stimulating hormone] level is indicated during the annual wellness check that patients on Medicare can get every year.

 

 

Q: Is TSH measured as part of routine blood tests?

A: No it’s not. It has to be ordered. I think that’s why we’re seeing this problem to begin with — because it’s not something we all have awareness about. I wasn’t aware myself that mildly high levels of thyroid could increase the risk of cognitive impairment. Certainly, I’m going to be much more aware in my practice.

Q: You also picked a study about silicosis in workers who are exposed to dust when they make engineered stone countertops, also known as quartz countertops. What were the findings?

A: Silicosis is a very serious lung condition that develops from exposure to crystalline silica. Essentially, sand gets inhaled into the lungs. Workers can be exposed when they’re making engineered stone countertops, the most popular countertops now in the United States.

This study is based on statewide surveys from 2019 to 2022 that the California Department of Public Health does routinely. They gathered cases of silicosis and found 52 — all men with an average age of 45. All but one were Latino immigrants, and most either had no insurance or very poor insurance.
 

Q: The study found that “diagnosis was delayed in 58%, with 38% presenting with advanced disease (progressive massive fibrosis), and 19% died.” What does that tell you?

A: It’s a very serious condition. Once it gets to the advanced stage, it will just continue to progress, and the person will die. That’s why it’s so important to know that it’s absolutely preventable.

Q: Is there a message here for internists?

A: If you treat a lot of immigrants or work in an area where there are a lot of industrial workers, you’re going to want to have a very high suspicion about it. If you see an atypical pattern on the chest x-ray or via diffusion scoring, have a low threshold for getting a pulmonary function test.

Doctors need to be aware and diagnose this very quickly. When patients present, you can pull them out of that work environment or put mitigation systems into place.
 

Q: California regulators were expected to put emergency rules into place in late December to protect workers. Did this study play a role in focusing attention on the problem?

A: This article, along with a commentary and podcast that we put out, really helped with advocacy to improve health and safety for workers at stone-cutting and fabrication shops.

Q: You were impressed by another study about airborne dangers, this one linking air pollution to dementia. What did researchers discover?

A: [This analysis] of more than 27,000 people in the Health and Retirement Study, a respected and rich database, found that exposure to air pollution was associated with greater rates of dementia — an increase of about 8% a year. Exposure to agricultural emissions and wildfire smoke were most robustly associated with a greater risk of dementia.

Q: How are these findings important, especially in light of the unhealthy air spawned by recent wildfires in the United States and Canada?

A: Studies like this will make it even more compelling that we are better prepared for air quality issues.

I grew up in Los Angeles, where smog and pollution were very big issues. I was constantly hearing about various mitigation strategies that were going into place. But after I moved to the East Coast, I almost never heard about prevention.

Now, I’m hoping we can keep this topic in the national conversation.
 

Q: You also highlighted a systematic review of the use of restraints in the emergency department. Why did you choose this research?

A: At JAMA Internal Medicine, we’re really focused on ways we can address health disparities and raise awareness of potential unconscious bias.

This review looked at 10 studies that included more than 2.5 million patient encounters, including 24,000 incidents of physical restraint use. They found that the overall rate of use of restraints was low at below 1%.

But when they are used, Black patients were 1.3 times more likely to be restrained than White patients.
 

Q: What’s the message here?

A: This is an important start to recognizing these differences and then changing our behavior. Perhaps restraints don’t need to be used as often in light of evidence, for example, of increased rates of misdiagnosis of psychosis in the Black population.

Q: How should physicians change their approach to restraints?

A: Restraints are not to be used to control disruption — wild behavior or verbal outbursts. They’re for when someone is a danger to themselves or others.

Dr. Inouye has no conflicts of interest.

Publications
Topics
Sections

Harvard Medical School’s Sharon K. Inouye, MD, MPH, is editor in chief of JAMA Internal Medicine and a leading voice in American gerontology. We asked her to choose five of the influential journal’s most impactful studies from 2023 and highlight important take-home messages for internists and their colleagues.
 

Q: One of the studies you chose suggests that the antiviral nirmatrelvir (Paxlovid) can ward off long COVID. Could you recap the findings?

A: Researchers followed a group of more than 280,000 Department of Veterans Affairs patients who were seen in 2022, had a positive COVID test, and had at least one risk factor for severe COVID. They focused on those who survived to 30 days after their COVID infection and compared those who received the drug within the first 5 days of a positive test with an equivalent control group.

They found that 13 long COVID symptoms were all significantly less common (relative risk = 0.74) in those who received nirmatrelvir. This was true no matter whether they’d ever had a COVID vaccination.
 

Q: How should this research affect clinical practice?

A: You can’t generalize from this to everyone because, of course, not everyone was included in this study. But it is highly suggestive that this drug is very effective for preventing long COVID.

Nirmatrelvir was touted as being able to shorten duration of illness and prevent hospitalization. But if you were low risk or you were already well into your COVID course, it wasn’t like rush, rush, rush to the doctor to get it.

This changes that equation because we know long COVID is such a huge issue. The vast majority of doctors who work with COVID patients and know this are now being more aggressive about prescribing it.
 

Q: What about patients whom the CDC considers to be at less risk — people with up-to-date vaccinations who are under 50 with mild-to-moderate COVID and no higher-risk medical conditions? Should they take nirmatrelvir?

A: The evidence is not 100% in yet. A study like this one needs to be repeated and include younger people without any risk factors to see if we see the same thing. So it’s a personal choice, and a personal calculus needs to be done. A lot of people are making that choice [to take the drug], and it can be a rational decision.

Q: You also chose a study that links high thyroid hormone levels to higher rates of dementia. What did it reveal?

A: This study looks at patients who had thyrotoxicosis — a thyroid level that’s too high — from hormone produced endogenously, and exogenously. Researchers tracked almost 66,000 patients aged 65 and older and found that thyrotoxicosis from all causes, whether it was endogenous or exogenous, was linked to an increased risk of dementia in a dose-response relationship (adjusted hazard ratio = 1.39).

Q: Is there a clinical take-home message here?

A: When we start patients on thyroid medication, they don’t always get reassessed on a regular basis. Given this finding, a TSH [thyroid-stimulating hormone] level is indicated during the annual wellness check that patients on Medicare can get every year.

 

 

Q: Is TSH measured as part of routine blood tests?

A: No it’s not. It has to be ordered. I think that’s why we’re seeing this problem to begin with — because it’s not something we all have awareness about. I wasn’t aware myself that mildly high levels of thyroid could increase the risk of cognitive impairment. Certainly, I’m going to be much more aware in my practice.

Q: You also picked a study about silicosis in workers who are exposed to dust when they make engineered stone countertops, also known as quartz countertops. What were the findings?

A: Silicosis is a very serious lung condition that develops from exposure to crystalline silica. Essentially, sand gets inhaled into the lungs. Workers can be exposed when they’re making engineered stone countertops, the most popular countertops now in the United States.

This study is based on statewide surveys from 2019 to 2022 that the California Department of Public Health does routinely. They gathered cases of silicosis and found 52 — all men with an average age of 45. All but one were Latino immigrants, and most either had no insurance or very poor insurance.
 

Q: The study found that “diagnosis was delayed in 58%, with 38% presenting with advanced disease (progressive massive fibrosis), and 19% died.” What does that tell you?

A: It’s a very serious condition. Once it gets to the advanced stage, it will just continue to progress, and the person will die. That’s why it’s so important to know that it’s absolutely preventable.

Q: Is there a message here for internists?

A: If you treat a lot of immigrants or work in an area where there are a lot of industrial workers, you’re going to want to have a very high suspicion about it. If you see an atypical pattern on the chest x-ray or via diffusion scoring, have a low threshold for getting a pulmonary function test.

Doctors need to be aware and diagnose this very quickly. When patients present, you can pull them out of that work environment or put mitigation systems into place.
 

Q: California regulators were expected to put emergency rules into place in late December to protect workers. Did this study play a role in focusing attention on the problem?

A: This article, along with a commentary and podcast that we put out, really helped with advocacy to improve health and safety for workers at stone-cutting and fabrication shops.

Q: You were impressed by another study about airborne dangers, this one linking air pollution to dementia. What did researchers discover?

A: [This analysis] of more than 27,000 people in the Health and Retirement Study, a respected and rich database, found that exposure to air pollution was associated with greater rates of dementia — an increase of about 8% a year. Exposure to agricultural emissions and wildfire smoke were most robustly associated with a greater risk of dementia.

Q: How are these findings important, especially in light of the unhealthy air spawned by recent wildfires in the United States and Canada?

A: Studies like this will make it even more compelling that we are better prepared for air quality issues.

I grew up in Los Angeles, where smog and pollution were very big issues. I was constantly hearing about various mitigation strategies that were going into place. But after I moved to the East Coast, I almost never heard about prevention.

Now, I’m hoping we can keep this topic in the national conversation.
 

Q: You also highlighted a systematic review of the use of restraints in the emergency department. Why did you choose this research?

A: At JAMA Internal Medicine, we’re really focused on ways we can address health disparities and raise awareness of potential unconscious bias.

This review looked at 10 studies that included more than 2.5 million patient encounters, including 24,000 incidents of physical restraint use. They found that the overall rate of use of restraints was low at below 1%.

But when they are used, Black patients were 1.3 times more likely to be restrained than White patients.
 

Q: What’s the message here?

A: This is an important start to recognizing these differences and then changing our behavior. Perhaps restraints don’t need to be used as often in light of evidence, for example, of increased rates of misdiagnosis of psychosis in the Black population.

Q: How should physicians change their approach to restraints?

A: Restraints are not to be used to control disruption — wild behavior or verbal outbursts. They’re for when someone is a danger to themselves or others.

Dr. Inouye has no conflicts of interest.

Harvard Medical School’s Sharon K. Inouye, MD, MPH, is editor in chief of JAMA Internal Medicine and a leading voice in American gerontology. We asked her to choose five of the influential journal’s most impactful studies from 2023 and highlight important take-home messages for internists and their colleagues.
 

Q: One of the studies you chose suggests that the antiviral nirmatrelvir (Paxlovid) can ward off long COVID. Could you recap the findings?

A: Researchers followed a group of more than 280,000 Department of Veterans Affairs patients who were seen in 2022, had a positive COVID test, and had at least one risk factor for severe COVID. They focused on those who survived to 30 days after their COVID infection and compared those who received the drug within the first 5 days of a positive test with an equivalent control group.

They found that 13 long COVID symptoms were all significantly less common (relative risk = 0.74) in those who received nirmatrelvir. This was true no matter whether they’d ever had a COVID vaccination.
 

Q: How should this research affect clinical practice?

A: You can’t generalize from this to everyone because, of course, not everyone was included in this study. But it is highly suggestive that this drug is very effective for preventing long COVID.

Nirmatrelvir was touted as being able to shorten duration of illness and prevent hospitalization. But if you were low risk or you were already well into your COVID course, it wasn’t like rush, rush, rush to the doctor to get it.

This changes that equation because we know long COVID is such a huge issue. The vast majority of doctors who work with COVID patients and know this are now being more aggressive about prescribing it.
 

Q: What about patients whom the CDC considers to be at less risk — people with up-to-date vaccinations who are under 50 with mild-to-moderate COVID and no higher-risk medical conditions? Should they take nirmatrelvir?

A: The evidence is not 100% in yet. A study like this one needs to be repeated and include younger people without any risk factors to see if we see the same thing. So it’s a personal choice, and a personal calculus needs to be done. A lot of people are making that choice [to take the drug], and it can be a rational decision.

Q: You also chose a study that links high thyroid hormone levels to higher rates of dementia. What did it reveal?

A: This study looks at patients who had thyrotoxicosis — a thyroid level that’s too high — from hormone produced endogenously, and exogenously. Researchers tracked almost 66,000 patients aged 65 and older and found that thyrotoxicosis from all causes, whether it was endogenous or exogenous, was linked to an increased risk of dementia in a dose-response relationship (adjusted hazard ratio = 1.39).

Q: Is there a clinical take-home message here?

A: When we start patients on thyroid medication, they don’t always get reassessed on a regular basis. Given this finding, a TSH [thyroid-stimulating hormone] level is indicated during the annual wellness check that patients on Medicare can get every year.

 

 

Q: Is TSH measured as part of routine blood tests?

A: No it’s not. It has to be ordered. I think that’s why we’re seeing this problem to begin with — because it’s not something we all have awareness about. I wasn’t aware myself that mildly high levels of thyroid could increase the risk of cognitive impairment. Certainly, I’m going to be much more aware in my practice.

Q: You also picked a study about silicosis in workers who are exposed to dust when they make engineered stone countertops, also known as quartz countertops. What were the findings?

A: Silicosis is a very serious lung condition that develops from exposure to crystalline silica. Essentially, sand gets inhaled into the lungs. Workers can be exposed when they’re making engineered stone countertops, the most popular countertops now in the United States.

This study is based on statewide surveys from 2019 to 2022 that the California Department of Public Health does routinely. They gathered cases of silicosis and found 52 — all men with an average age of 45. All but one were Latino immigrants, and most either had no insurance or very poor insurance.
 

Q: The study found that “diagnosis was delayed in 58%, with 38% presenting with advanced disease (progressive massive fibrosis), and 19% died.” What does that tell you?

A: It’s a very serious condition. Once it gets to the advanced stage, it will just continue to progress, and the person will die. That’s why it’s so important to know that it’s absolutely preventable.

Q: Is there a message here for internists?

A: If you treat a lot of immigrants or work in an area where there are a lot of industrial workers, you’re going to want to have a very high suspicion about it. If you see an atypical pattern on the chest x-ray or via diffusion scoring, have a low threshold for getting a pulmonary function test.

Doctors need to be aware and diagnose this very quickly. When patients present, you can pull them out of that work environment or put mitigation systems into place.
 

Q: California regulators were expected to put emergency rules into place in late December to protect workers. Did this study play a role in focusing attention on the problem?

A: This article, along with a commentary and podcast that we put out, really helped with advocacy to improve health and safety for workers at stone-cutting and fabrication shops.

Q: You were impressed by another study about airborne dangers, this one linking air pollution to dementia. What did researchers discover?

A: [This analysis] of more than 27,000 people in the Health and Retirement Study, a respected and rich database, found that exposure to air pollution was associated with greater rates of dementia — an increase of about 8% a year. Exposure to agricultural emissions and wildfire smoke were most robustly associated with a greater risk of dementia.

Q: How are these findings important, especially in light of the unhealthy air spawned by recent wildfires in the United States and Canada?

A: Studies like this will make it even more compelling that we are better prepared for air quality issues.

I grew up in Los Angeles, where smog and pollution were very big issues. I was constantly hearing about various mitigation strategies that were going into place. But after I moved to the East Coast, I almost never heard about prevention.

Now, I’m hoping we can keep this topic in the national conversation.
 

Q: You also highlighted a systematic review of the use of restraints in the emergency department. Why did you choose this research?

A: At JAMA Internal Medicine, we’re really focused on ways we can address health disparities and raise awareness of potential unconscious bias.

This review looked at 10 studies that included more than 2.5 million patient encounters, including 24,000 incidents of physical restraint use. They found that the overall rate of use of restraints was low at below 1%.

But when they are used, Black patients were 1.3 times more likely to be restrained than White patients.
 

Q: What’s the message here?

A: This is an important start to recognizing these differences and then changing our behavior. Perhaps restraints don’t need to be used as often in light of evidence, for example, of increased rates of misdiagnosis of psychosis in the Black population.

Q: How should physicians change their approach to restraints?

A: Restraints are not to be used to control disruption — wild behavior or verbal outbursts. They’re for when someone is a danger to themselves or others.

Dr. Inouye has no conflicts of interest.

Publications
Publications
Topics
Article Type
Sections
Disallow All Ads
Content Gating
No Gating (article Unlocked/Free)
Alternative CME
Disqus Comments
Default
Use ProPublica
Hide sidebar & use full width
render the right sidebar.
Conference Recap Checkbox
Not Conference Recap
Clinical Edge
Display the Slideshow in this Article
Medscape Article
Display survey writer
Reuters content
Disable Inline Native ads
WebMD Article